Desigualdades matemáticas clásicas · 2018-02-27 · 3 Resumen En esta memoria recogemos algunas...

73
Cristina Bermejo Moreno José Luis Ansorena Barasoain Facultad de Ciencia y Tecnología Grado en Matemáticas 2016-2017 Título Director/es Facultad Titulación Departamento TRABAJO FIN DE GRADO Curso Académico Desigualdades matemáticas clásicas Autor/es

Transcript of Desigualdades matemáticas clásicas · 2018-02-27 · 3 Resumen En esta memoria recogemos algunas...

Page 1: Desigualdades matemáticas clásicas · 2018-02-27 · 3 Resumen En esta memoria recogemos algunas de las desigualdades m as importantes en Matem aticas, sus or ge-nes y aplicaciones.

Cristina Bermejo Moreno

José Luis Ansorena Barasoain

Facultad de Ciencia y Tecnología

Grado en Matemáticas

2016-2017

Título

Director/es

Facultad

Titulación

Departamento

TRABAJO FIN DE GRADO

Curso Académico

Desigualdades matemáticas clásicas

Autor/es

Page 2: Desigualdades matemáticas clásicas · 2018-02-27 · 3 Resumen En esta memoria recogemos algunas de las desigualdades m as importantes en Matem aticas, sus or ge-nes y aplicaciones.

© El autor© Universidad de La Rioja, Servicio de Publicaciones,

publicaciones.unirioja.esE-mail: [email protected]

Desigualdades matemáticas clásicas, trabajo fin de grado de Cristina BermejoMoreno, dirigido por José Luis Ansorena Barasoain (publicado por la Universidad de LaRioja), se difunde bajo una Licencia Creative Commons Reconocimiento-NoComercial-

SinObraDerivada 3.0 Unported. Permisos que vayan más allá de lo cubierto por esta licencia pueden solicitarse a los

titulares del copyright.

Page 3: Desigualdades matemáticas clásicas · 2018-02-27 · 3 Resumen En esta memoria recogemos algunas de las desigualdades m as importantes en Matem aticas, sus or ge-nes y aplicaciones.

Facultad de Ciencia y Tecnología

TRABAJO FIN DE GRADO

Grado en Matemáticas

Desigualdades matemáticas clásicas

Alumno:

Cristina Bermejo Moreno

Tutores:

José Luis Ansorena Barasoain

Logroño, 23 de junio de 2017

Page 4: Desigualdades matemáticas clásicas · 2018-02-27 · 3 Resumen En esta memoria recogemos algunas de las desigualdades m as importantes en Matem aticas, sus or ge-nes y aplicaciones.
Page 5: Desigualdades matemáticas clásicas · 2018-02-27 · 3 Resumen En esta memoria recogemos algunas de las desigualdades m as importantes en Matem aticas, sus or ge-nes y aplicaciones.

TRABAJO FIN DE GRADO

GRADO EN MATEMATICAS

DESIGUALDADES MATEMATICASCLASICAS

Cristina Bermejo Moreno

dirigido por

Jose Luis Ansorena Barasoain

Universidad de La Rioja

Facultad de Ciencia y Tecnologıa

Departamento de Matematicas y Computacion

Curso 2016/17

Page 6: Desigualdades matemáticas clásicas · 2018-02-27 · 3 Resumen En esta memoria recogemos algunas de las desigualdades m as importantes en Matem aticas, sus or ge-nes y aplicaciones.
Page 7: Desigualdades matemáticas clásicas · 2018-02-27 · 3 Resumen En esta memoria recogemos algunas de las desigualdades m as importantes en Matem aticas, sus or ge-nes y aplicaciones.

3

Resumen

En esta memoria recogemos algunas de las desigualdades mas importantes en Matematicas, sus orıge-nes y aplicaciones. En concreto, nos centramos en el estudio de la desigualdad de Jensen, de medias,Cauchy-Schwarz, Holder, de reordenamiento, Chebyshev, Muirhead y Schur.

Para el estudio de cada desigualdad, hemos dividido el trabajo en ocho capıtulos. Cada capıtulo constade dos partes bien diferenciadas: una primera parte teorica y una segunda, practica. Comenzaremos conel enunciado y demostracion de teoremas y resultados relativos a la desigualdad a tratar. En la medida delo posible, buscaremos combinar demostraciones matematicamente sencillas con otras que requieren unmayor nivel de abstraccion y complejidad. Seguidamente, resolveremos ejercicios y problemas extraıdos,principalmente, de la Olimpiada Matematica Espanola y la Olimpiada Matematica Internacional (IMOpor sus siglas en ingles). Cabe destacar que la IMO es la mas antigua, y pionera, de las OlimpiadasInternacionales de Ciencias.

Veremos, ademas, algunas interesantes aplicaciones practicas de estas desigualdades.

Hemos incluido, siempre que ha sido posible, notas biograficas sobre los matematicos que descubrieronlas desigualdades o fueron pioneros en su uso y que, en muchos casos, les dieron su nombre.

Page 8: Desigualdades matemáticas clásicas · 2018-02-27 · 3 Resumen En esta memoria recogemos algunas de las desigualdades m as importantes en Matem aticas, sus or ge-nes y aplicaciones.
Page 9: Desigualdades matemáticas clásicas · 2018-02-27 · 3 Resumen En esta memoria recogemos algunas de las desigualdades m as importantes en Matem aticas, sus or ge-nes y aplicaciones.

5

Abstract

In this report we gather some of the most important inequalities in Mathematics, including theirorigins and applications. In particular, we focus on the study of Jensen’s inequality, the generalized meaninequality, Cauchy-Schwarz inequality, Holder’s inequality, the rearrangement inequality, Chebyshev’sinequality, Muirhead’s inequality and Schur’s inequality.

In order to isolate the study of each inequality, we have divided this work into eight chapters. Eachchapter is devoted to one inequality and consists of two different parts: the theoretical one and the practi-cal one. We will start with the statement and proof of some theorems and results related to the inequalitywe are dealing with. We emphasize that, as far as possible, we will try to combine simple mathematicalproofs with others requiring a higher level of abstraction and complexity. Straightaway, we will solve someexercises and problems mainly extracted from the Spanish Mathematical Olympiad and the InternationalMathematical Olympiad (IMO). It should be pointed out that the IMO is a pioneer scientific contest andthe most ancient one of all the International Science Olympiads.

Besides, we will provide the reader with some interesting practical applications of these inequalities.

We have included, whenever possible, some biographical notes about the mathematicians who, eitherdiscovered the inequalities or were pioneers in their use. In many cases, these inequalities are named afterthem.

Page 10: Desigualdades matemáticas clásicas · 2018-02-27 · 3 Resumen En esta memoria recogemos algunas de las desigualdades m as importantes en Matem aticas, sus or ge-nes y aplicaciones.
Page 11: Desigualdades matemáticas clásicas · 2018-02-27 · 3 Resumen En esta memoria recogemos algunas de las desigualdades m as importantes en Matem aticas, sus or ge-nes y aplicaciones.

7

Indice general

Resumen 3

Abstract 5

Introduccion 11

1. La desigualdad de Jensen 13

1.1. Introduccion . . . . . . . . . . . . . . . . . . . . . . . . . . . . . . . . . . . . . . . . . . . . 13

1.2. La desigualdad de Jensen discreta . . . . . . . . . . . . . . . . . . . . . . . . . . . . . . . 17

1.3. La desigualdad de Jensen en la teorıa de integracion . . . . . . . . . . . . . . . . . . . . . 19

1.4. Problemas . . . . . . . . . . . . . . . . . . . . . . . . . . . . . . . . . . . . . . . . . . . . . 21

2. La desigualdad de medias 25

2.1. Enunciado y demostracion . . . . . . . . . . . . . . . . . . . . . . . . . . . . . . . . . . . . 25

2.2. Problemas . . . . . . . . . . . . . . . . . . . . . . . . . . . . . . . . . . . . . . . . . . . . . 30

3. La desigualdad de Cauchy-Schwarz 35

3.1. Augustin Louis Cauchy . . . . . . . . . . . . . . . . . . . . . . . . . . . . . . . . . . . . . 35

3.2. Karl Hermann Schwarz . . . . . . . . . . . . . . . . . . . . . . . . . . . . . . . . . . . . . . 36

3.3. Viktor Yakovlevich Bunyakovsky . . . . . . . . . . . . . . . . . . . . . . . . . . . . . . . . 36

3.4. Introduccion . . . . . . . . . . . . . . . . . . . . . . . . . . . . . . . . . . . . . . . . . . . . 37

3.5. La desigualdad de Cauchy-Schwarz . . . . . . . . . . . . . . . . . . . . . . . . . . . . . . . 38

3.6. La desigualdad de Cauchy-Schwarz en la teorıa de integracion . . . . . . . . . . . . . . . . 39

3.7. La desigualdad de Cauchy-Schwarz para sumas . . . . . . . . . . . . . . . . . . . . . . . . 39

3.8. El Lema de Titu . . . . . . . . . . . . . . . . . . . . . . . . . . . . . . . . . . . . . . . . . 40

3.9. La desigualdad triangular . . . . . . . . . . . . . . . . . . . . . . . . . . . . . . . . . . . . 41

3.10. Problemas . . . . . . . . . . . . . . . . . . . . . . . . . . . . . . . . . . . . . . . . . . . . . 43

4. La desigualdad de Holder 47

4.1. Introduccion . . . . . . . . . . . . . . . . . . . . . . . . . . . . . . . . . . . . . . . . . . . . 47

4.2. Enunciado y demostracion . . . . . . . . . . . . . . . . . . . . . . . . . . . . . . . . . . . . 48

5. La desigualdad de reordenamiento 51

5.1. Introduccion y enunciado . . . . . . . . . . . . . . . . . . . . . . . . . . . . . . . . . . . . 51

5.2. Demostracion . . . . . . . . . . . . . . . . . . . . . . . . . . . . . . . . . . . . . . . . . . . 51

5.3. Problemas . . . . . . . . . . . . . . . . . . . . . . . . . . . . . . . . . . . . . . . . . . . . . 53

6. La desigualdad de Chebyshev 55

6.1. Desigualdad de Chebyshev sobre sumas . . . . . . . . . . . . . . . . . . . . . . . . . . . . 56

6.2. Problemas . . . . . . . . . . . . . . . . . . . . . . . . . . . . . . . . . . . . . . . . . . . . . 57

7. Desigualdad de Muirhead 59

7.1. Introduccion . . . . . . . . . . . . . . . . . . . . . . . . . . . . . . . . . . . . . . . . . . . . 59

7.2. El Teorema de Muirhead . . . . . . . . . . . . . . . . . . . . . . . . . . . . . . . . . . . . . 60

7.3. Problemas . . . . . . . . . . . . . . . . . . . . . . . . . . . . . . . . . . . . . . . . . . . . . 62

Page 12: Desigualdades matemáticas clásicas · 2018-02-27 · 3 Resumen En esta memoria recogemos algunas de las desigualdades m as importantes en Matem aticas, sus or ge-nes y aplicaciones.

8 INDICE GENERAL

8. La desigualdad de Schur 658.1. Introduccion . . . . . . . . . . . . . . . . . . . . . . . . . . . . . . . . . . . . . . . . . . . . 658.2. Enunciado y demostracion . . . . . . . . . . . . . . . . . . . . . . . . . . . . . . . . . . . . 658.3. Problemas . . . . . . . . . . . . . . . . . . . . . . . . . . . . . . . . . . . . . . . . . . . . . 66

Conclusion 67

Bibliografıa 69

Page 13: Desigualdades matemáticas clásicas · 2018-02-27 · 3 Resumen En esta memoria recogemos algunas de las desigualdades m as importantes en Matem aticas, sus or ge-nes y aplicaciones.

Le Calcul infinitesimal, [...], est l’apprentissage du maniement des inegalites bien

plus que des egalites, et on pourrait le resumer en trois mot: majorer, minorer,

approcher.

Jean Dieudonne, Calcul Infinitesimal, (1968)

Page 14: Desigualdades matemáticas clásicas · 2018-02-27 · 3 Resumen En esta memoria recogemos algunas de las desigualdades m as importantes en Matem aticas, sus or ge-nes y aplicaciones.
Page 15: Desigualdades matemáticas clásicas · 2018-02-27 · 3 Resumen En esta memoria recogemos algunas de las desigualdades m as importantes en Matem aticas, sus or ge-nes y aplicaciones.

11

Introduccion

En traduccion libre, la nota del matematico frances del grupo Bourbaki Jean Dieudonne, extraıda desu manual de Calculo Infinitesimal [4], dice que “el Calculo Infinitesimal consiste en aprender a manejarlas desigualdades tan bien como las igualdades, y lo podrıamos resumir en tres palabras: mayorar, mi-norar, aproximar”. Tres palabras pueden parecer pocas para describir que es el Calculo Infinitesimal y,por extension, el Analisis Matematico. Sin embargo, minorar y mayorar son dos divisiones de la mismaaccion (acotar), y aproximar es acotar por numeros reales positivos arbitrariamente pequenos. Ası que,permitidnos resumir el Analisis Matematico en la accion de acotar. Y, como Jean Dieudonne remarca, labase para acotar es manejar desigualdades.

En este trabajo pretendemos presentar, demostrar y utilizar algunas de las desigualdades matemati-cas clasicas mas sencillas y conocidas. Muchas de estas desigualdades, aunque pueden enunciarse comosencillas desigualdades numericas, pueden tambien interpretarse en el contexto de la Teorıa de integra-cion abstracta o, incluso, del Analisis Funcional. En estos casos, incluiremos diferentes enunciados ydemostraciones. De esta manera, pretendemos que el manuscrito tenga diferentes lecturas y sea util tantopara lectores de un nivel preuniversitario como para Graduados en Matematicas. Asimismo hemos in-tentado, siempre que nos ha sido posible, buscar las raıces historicas de las desigualdades y de sus autores.

Las demostraciones de los resultados y aplicaciones que daremos estan, en muchos casos, extraıdas delibros de problemas orientados a concursos matematicos, fundamentalmente de la Olimpiada MatematicaEspanola y de la Olimpiada Matematica Internacional ([5, 3, 1]) y de recursos electronicos ([7, 8, 10, 11]).Algunas ideas son tambien propias. Las referencias basicas para los enunciados y demostraciones desde elpunto de vista de la Teorıa de integracion abstracta han sido los textos clasicos de Analisis Matematico[6] y [9].

Page 16: Desigualdades matemáticas clásicas · 2018-02-27 · 3 Resumen En esta memoria recogemos algunas de las desigualdades m as importantes en Matem aticas, sus or ge-nes y aplicaciones.
Page 17: Desigualdades matemáticas clásicas · 2018-02-27 · 3 Resumen En esta memoria recogemos algunas de las desigualdades m as importantes en Matem aticas, sus or ge-nes y aplicaciones.

13

Capıtulo 1

La desigualdad de Jensen

Johan Ludwig William Valdemar Jensen (Nakskov, 8 de mayo de 1859 - Copenhague, 5 de marzode 1925), mas conocido como Johan Jensen, fue un matematico e ingeniero danes. Con 17 anos fueinscrito en el Colegio de Tecnologıa de Copenhague, ciudad en la que anos mas tarde se convirtio enrespetado ingeniero de la Companıa Telefonica, llegando a ocupar el puesto de Jefe de la Oficina Tecnicade Investigacion (1890). Aunque estudio Matematicas en la Universidad, su conocimiento a nivel avanzadode esta materia lo logro de manera autodidacta. Todos sus trabajos matematicos los llevo a cabo en sutiempo libre. Publico un trabajo de investigacion y presidio la Sociedad Matematica Danesa de 1892 a1903.

Ha pasado a la historia por ser la primera persona que se intereso por un fenomeno caracterıstico delas funciones continuas no lineales (concavas o convexas), conocido como Desigualdad de Jensen. Estapropiedad aparece en multiples contextos y tiene diversas aplicaciones en ambitos como el financiero o elsanitario.

1.1. Introduccion

Sea I = (a, b) un intervalo de numeros reales, ϕ una funcion definida sobre I con valores en R. Decimosque ϕ es convexa si

ϕ((1− λ)x+ λy) ≤ (1− λ)ϕ(x) + λϕ(y) para todo x, y ∈ I, 0 ≤ λ ≤ 1. (1.1)

En caso de que la desigualdad en (1.1) sea estricta, excepto en los casos triviales en que x = y o λ ∈ 0, 1,decimos que f es estrictamente convexa. Cuando la desigualdad que obtenemos en (1.1) es la contraria,decimos que f es concava. O sea, ϕ es concava si y solo si −ϕ es convexa. De manera similar definimoscuando una funcion es estrictamente concava.

Si una funcion es convexa en (a, b) entonces se tiene que

ϕ(t)− ϕ(s)

t− s≤ ϕ(v)− ϕ(u)

v − u, s < t, u < v, s ≤ u, t ≤ v. (1.2)

Page 18: Desigualdades matemáticas clásicas · 2018-02-27 · 3 Resumen En esta memoria recogemos algunas de las desigualdades m as importantes en Matem aticas, sus or ge-nes y aplicaciones.

14 CAPITULO 1. LA DESIGUALDAD DE JENSEN

De nuevo, para ϕ estrictamente convexa, las desigualdades son estrictas excepto en los casos triviales.Esta desigualdad significa que si una funcion es convexa, las ratio de crecimiento son crecientes. Dehecho, la implicacion contraria es cierta. Si se cumple que

ϕ(t)− ϕ(s)

t− s≤ ϕ(v)− ϕ(s)

v − s, s ≤ t < v, (1.3)

la funcion es convexa (estrictamente convexa si la desigualdad es estricta).De (1.2) se deduce que toda funcion convexa es continua y que para todo a < x < b existen las

derivadas laterales

ϕ′(x−) = lımy→x−

ϕ(y)− ϕ(x)

y − x= supa<t<s≤x

ϕ(y)− ϕ(x)

y − x(1.4)

ϕ′(x+) = lımy→x+

ϕ(y)− ϕ(x)

y − x= ınfy≤t<s<b

ϕ(y)− ϕ(x)

y − x(1.5)

y, ademas,

ϕ′(x−) ≤ ϕ′(x+) ≤ ϕ′(y−) ≤ ϕ′(y+), a < x < y < b, (1.6)

donde ϕ′(x+) < ϕ′(y−) si ϕ es estrictamente convexa.Naturalmente, una funcion convexa no tiene por que ser derivable en todos sus puntos (la funcion

| · | es un importante ejemplo). Sin embargo, (1.6) conduce de modo sencillo a caracterizaciones de laconvexidad de funciones derivables.

Teorema 1. Sea ϕ : (a, b)→ R derivable.

ϕ es convexa si y solo si ϕ′ es creciente.

ϕ es estrictamente convexa si y solo si ϕ′ es estrictamente creciente.

Teorema 2. Sea ϕ : I = (a, b)→ R dos veces derivable.

ϕ es convexa si y solo si ϕ′′(t) ≥ 0 para todo t ∈ I.

ϕ es estrictamente convexa si y solo si ϕ′′ > 0 excepto quizas en un conjunto de interior vacıo.

Comentario. Se tienen las siguientes propiedades elementales relativas a la convexidad:

Si f es una funcion convexa entonces la funcion opuesta −f es concava, y viceversa.

Si f y g son funciones convexas entonces f + g sigue siendo una funcion convexa.

Si f es estrictamente convexa en (0, a) entonces x 7→ f(a− x) es estrictamente convexa en (0, a).

Si f es estrictamente convexa en (a, b) y c ∈ R entonces x 7→ c+ f(x) es estrictamente convexa en(a, b).

Un poco mas elaborada es la demostracion de la siguiente propiedad.

Lema 1. Sea f : I → J una funcion convexa, creciente y biyectiva. Entonces su inversa f−1 es concava.

Demostracion. Sea f : I → J biyectiva, x, y ∈ J , λ ∈ [0, 1]. Probar que

f−1((1− λ)x+ λy) ≥ f(1− λ)f−1(x) + λf−1(y)

equivale a probar que

(1− λ)x+ λy ≥ f((1− λ)f−1(x) + λf−1(y)).

Utilizando la convexidad de f ,

f((1− λ)f−1(x) + λf−1(y)) ≤ (1− λ)f(f−1(x)) + λf(f−1(y)) = (1− λ)x+ λy.

Page 19: Desigualdades matemáticas clásicas · 2018-02-27 · 3 Resumen En esta memoria recogemos algunas de las desigualdades m as importantes en Matem aticas, sus or ge-nes y aplicaciones.

1.1. INTRODUCCION 15

Comentario. Vamos a estudiar la convexidad o concavidad de las funciones x−1, ex, sin(x), xa cona ≥ 1 y log(x), que apareceran a lo largo de este trabajo. Veamos una breve demostracion de cada unade ellas (en el caso de ser posible, sin usar calculo diferencial).

La funcion x 7→ 1/x es estrictamente convexa en (0,∞).

Demostracion. Probamos la convexidad de la funcion sin usar calculo diferencial, notamos que laconvexidad es equivalente a la desigualdad

1

(1− λ)x+ λy≤ (1− λ)

1

x+ λ

1

y, 0 < x, 0 < y, 0 ≤ λ ≤ 1,

que a su vez equivale a

1 ≤ (1− λ)(1− λ)x+ λy

x+ λ

(1− λ)x+ λy

y, 0 < x, 0 < y, 0 ≤ λ ≤ 1.

Tambien es equivalente

1 ≤ (1− λ)2 + λ2 + λ(1− λ)

(y

x+x

y

), 0 < x, 0 < y, 0 ≤ λ ≤ 1,

que a su vez equivale a

2λ(1− λ) ≤ λ(1− λ)

(y

x+x

y

), 0 < x, 0 < y, 0 ≤ λ ≤ 1.

El resultado se obtiene combinando la desigualdad λ(1− λ) ≥ 0 para todo 0 ≤ λ ≤ 1, con igualdadpara λ ∈ 0, 1, con la desigualdad 2 ≤ x/y + y/x para todo x, y > 0, con igualdad cuando x = y(ver Lema 2).

La funcion exponencial x 7→ ex es estrictamente convexa.

Demostracion. La funcion exponencial se define como

exp(z) =∞∑n=0

1

n!zn.

Esta expresion nos da facilmente que exp′′ = exp > 0. Tambien es claro que exp es una funcion realde variable real y que exp(x) > 0 si x > 0. Puesto que exp(z) · exp(w) = exp(z + w) (no incluimosla demostracion de este hecho) se tiene que exp(z) · exp(−z) = 1 y, por tanto exp(x) > 0 para todox ≤ 0.

Incluimos tambien una demostracion basada en la idea intuitiva de potenciacion. Remarcamos que,sin recurrir a tecnicas de funciones analıticas o series de potencias, la potenciacion de base unnumero positivo, que debe cumplir las propiedades

a0 = 1, a > 0, (1.7)

ax+y = axay, a > 0, x, y ∈ R, (1.8)

(ax)y = axy, a > 0, x, y ∈ R, (1.9)

puede definirse de la siguiente manera:

• an, con a > 0 y n ∈ N es el producto de a consigo mismo n veces.

• a1/n, con a > 0 y n ∈ N es la raız n-esima positiva de a.

• La igualdad (1.9) nos determina el valor de ax con a > 0 y x racional positivo.

Page 20: Desigualdades matemáticas clásicas · 2018-02-27 · 3 Resumen En esta memoria recogemos algunas de las desigualdades m as importantes en Matem aticas, sus or ge-nes y aplicaciones.

16 CAPITULO 1. LA DESIGUALDAD DE JENSEN

• Ahora las igualdades (1.7) y (1.9) nos determinan el valor de ax con a > 0 y x racionalcualquiera.

• Puesto que los racionales son densos en los reales, definimos ax, con a > 0 y x ∈ R, poraproximacion.

Probemos ahora la convexidad de las funciones ϕa : R → R dadas por ϕa(x) = ax. Gracias a (1.3)es suficiente probar que

at − as

t− s<av − as

v − s, a > 0, s ≤ t < v.

Multiplicando por a−s y considerando t− s = x, v − s = y, basta probar

ax − 1

x<ay − 1

y, a > 0, 0 ≤ x < y.

Por aproximacion, es suficiente considerar x e y racionales. Por tanto, escribiendo x = m/k, y = n/kcon k, m, n enteros positivos, y multiplicando por 1/k basta probar,

am/k − 1

m<an/k − 1

n, a > 0, 0 ≤ m < n, m, n ∈ Z.

Tomando ahora b = a1/k basta probar

bm − 1

m<bn − 1

n, b > 0, 0 ≤ m < n, m, n ∈ Z.

Por el principio de induccion, basta considerar el caso m = n− 1. Por tanto basta probar que

(n− 1)(bn − 1)− n(bn−1 − 1) > 0, b > 0, n ∈ N.

Para este fin realizamos la manipulacion

(n− 1)(bn − 1)− n(bn−1 − 1) = n(bn − bn−1)− (bn − 1) = (b− 1)

(nbn−1 −

n−1∑k=0

bk

)> 0.

La funcion x 7→ sin(x) es estrictamente concava en el intervalo (0, π).

Demostracion. La manera mas precisa de demostrar este hecho es utilizar la definicion de la funcionseno, que se realiza a partir de la funcion exponencial. Tenemos que

sin(z) =eiz − e−iz

2i,

de donde se deduce facilmente que sin es una funcion real de variable real. Demostrar que sin(x) > 0para 0 < x < π requiere algo de tecnica (para hacerlo debemos definir π como el doble del menorvalor positivo en el que se anula la funcion coseno). Pero, si damos esto por bueno, la concavidadestricta es consecuencia del hecho sin′′ = − sin 0.No obstante, tambien es posible una demostracion de la concavidad para numeros reales entre 0 yπ (o angulos en el primer y segundo cuadrante), basada en la definicion intuitiva y geometrica dela funcion sin: el seno de un angulo x es la distancia entre el eje de abcisas y el punto de corte entrela semirrecta que determina el angulo y la circunferencia unidad. Incluimos una demostracion sinpalabras.

Page 21: Desigualdades matemáticas clásicas · 2018-02-27 · 3 Resumen En esta memoria recogemos algunas de las desigualdades m as importantes en Matem aticas, sus or ge-nes y aplicaciones.

1.2. LA DESIGUALDAD DE JENSEN DISCRETA 17

La funcion x 7→ xa, a ≥ 1 es convexa.

Demostracion. La convexidad es equivalente a la desigualdad

((1− λ)x+ λy)a ≤ (1− λ)xa + λya

Llamamos y/x = u (el caso x = 0 es sencillo) y nos queda

((1− λ) + λu)a ≤ (1− λ) + λua

Por tanto, basta probar esta ultima, que puede escribirse de la forma

(1 + λ(u− 1))a ≤ 1 + λ(ua − 1)

y sabemos que es cierta por el Lema 2.1. La igualdad se da cuando u = 1, o sea x = y.

La funcion log : (0,∞) → (−∞,∞) es concava. Este hecho es consecuencia de la convexidad de lafuncion exponencial.

1.2. La desigualdad de Jensen discreta

En muchas ocasiones, sobre todo en aplicaciones a la resolucion de problemas, es suficiente con utilizaruna version discreta de la desigualdad de Jensen. Por ese motivo, y en nuestro intento de incluir en nuestrasnotas demostraciones que un alumno preuniversitario o recien llegado a la Universidad pueda entender,comenzamos con una demostracion elemental de esta version de la desigualdad.

Teorema 3 (Desigualdad de Jensen, version discreta). Sea f : (a, b) → R una funcion convexa. Sean ∈ N, n ≥ 2. Dados numeros λk ≥ 0, a < xk < b (k = 1, . . . , n) tales que

∑nk=1 λk = 1, entonces,

f

(n∑k=1

λkxk

)≤

n∑k=1

λkf(xk). (1.10)

En caso de que ϕ sea estrictamente convexa la desigualdad es estricta, excepto en los casos triviales enlos que exista k tal que λk = 1 o existe x tal que xk = x para todo k = 1, . . . , n.

Para una funcion concava se obtiene un resultado similar, cambiando el sentido de la desigualdad.

Demostracion. En caso de que exista k tal que λk = 1, entonces λj = 0 para j 6= k y es evidente que severifica la igualdad en (1.10).

En caso de que exista x tal que xk = x para todo k = 1, . . . n, entonces∑nk=1 λkxk = x y es evidente

que se cumple la igualdad en (1.10).

Page 22: Desigualdades matemáticas clásicas · 2018-02-27 · 3 Resumen En esta memoria recogemos algunas de las desigualdades m as importantes en Matem aticas, sus or ge-nes y aplicaciones.

18 CAPITULO 1. LA DESIGUALDAD DE JENSEN

Tras estas consideraciones previas, basta probar la desigualdad en el caso en que λk > 0 para todok = 1, . . . , n. Lo haremos por el metodo de induccion.

• Base de induccion. Para n = 2 la desigualdad del enunciado es

f(λ1x1 + λ2x2) ≤ λ1f(x1) + λ2f(x2)

donde λ1 y λ2 son numeros positivos con λ1 + λ2 = 1. Tomando x1 = x, x2 = y, λ1 = λ, observamos queesta desigualdad es exactamente la de la definicion de funcion convexa (1.1). Explıcitamente,

f(λx+ (1− λ)y) ≤ λf(x) + (1− λ)f(y),

f(λ1x1 + (1− λ1)x2) ≤ λ1f(x1) + (1− λ1)f(x2),

f(λ1x+ λ2y) ≤ λ1f(x) + λ2f(y).

Igualmente, que la desigualdad sea estricta, excepto en los casos triviales, es consecuencia de la definicionde convexidad estricta, por lo que se cumple la base de induccion.

• Paso de induccion. Supongamos que la desigualdad (1.10) es cierta para un numero natural n ≥ 2y probemos que, en tal caso, tambien es cierta para n+ 1.

Sean λk > 0, a < xk < b (k = 1, . . . n+ 1) tales que∑n+1k=1 λk = 1. Ya hemos remarcado que λn+1 6= 1.

Consideramos

αk =λk

1− λn+1> 0, k = 1, . . . , n

x =n∑k=1

αkxk.

Tenemos quen∑k=1

αk =1

1− λk+1

n∑k=1

λk =1

1− λk+1(1− λk+1) = 1.

Por tanto, el numero x pertenece a I = (a, b) porque esta comprendido entre el maximo y el mınimo delos numeros xk, 1 ≤ k ≤ n. Ademas,

n+1∑k=1

λkxk = (1− λn+1)n∑k=1

λk1− λn+1

xk + λn+1xn+1 = (1− λn+1)x+ λn+1xn+1.

Usando que f es convexa, tenemos que

f

(n+1∑k=1

λkxk

)≤ (1− λn+1)f(x) + λn+1f(xn+1).

La hipotesis de induccion aplicada a x =∑nk=1 αkxk con αk > 0 y

∑nk=1 αk = 1 nos da

f(x) ≤n∑k=1

αkf(xk) =n∑k=1

λk1− λn+1

f(xk).

Combinando las dos ultimas desigualdades se deduce que

f

(n+1∑k=1

λkxk

)≤ (1− λn+1)

n∑k=1

λk1− λn+1

f(xk) + λn+1f(xn+1) =n+1∑k=1

λkf(xk).

Ademas, si ϕ es estrictamente convexa, y no tenemos una desigualdad estricta, ambas desigualdadesdeben ser igualdades. Por hipotesis de induccion, xk = x (k = 1, . . . , n). Por definicion de convexidadestricta, x = xn+1.

Page 23: Desigualdades matemáticas clásicas · 2018-02-27 · 3 Resumen En esta memoria recogemos algunas de las desigualdades m as importantes en Matem aticas, sus or ge-nes y aplicaciones.

1.3. LA DESIGUALDAD DE JENSEN EN LA TEORIA DE INTEGRACION 19

1.3. La desigualdad de Jensen en la teorıa de integracion

Damos a continuacion un enunciado abstracto (en terminos de integrales respecto de medidas de pro-babilidad) de la desigualdad de Jensen. Antes de ello, notamos que para una funcion convexa ϕ : (a, b)→ Rexisten los lımites laterales.

ϕ(a+) := lımt→a+

ϕ(t), ϕ(b−) := lımt→b−

ϕ(t). (1.11)

En consecuencia, ϕ se extiende de manera natural a una funcion definida en [a, b] con valores en [−∞,∞].El enunciado que sigue debemos entenderlo en este sentido.

Teorema 4 (Desigualdad de Jensen). Sea µ una medida positiva sobre una σ-algebra Σ en un conjunto Ωtal que µ(Ω) = 1. Sea ϕ : (a, b)→ R una funcion convexa. Sea f : Ω→ R Σ-medible tal que a ≤ f(ω) ≤ bpara todo ω ∈ Ω. Entonces

ϕ

(∫Ω

f dµ

)≤∫

Ω

(ϕ f) dµ, (1.12)

siempre que ambas integrales en la desigualdad tengan sentido. En caso de que ϕ sea estrictamenteconvexa, la desigualdad es estricta excepto en los siguientes casos:

El caso trivial en el que f es constante µ-c.t.p.

b =∫

Ωf dµ =∞ = ϕ(∞).

a =∫

Ωf dµ = −∞, ϕ(−∞) =∞.

b =∫

Ωf dµ =∞,

∫Ωϕ f dµ = −∞.

a =∫

Ωf dµ = −∞,

∫Ωϕ f dµ = −∞.

Nota. En el Teorema 4 admitimos la posibilidad de que tanto la funcion f como la media∫

Ωf dµ

tomen valores en los extremos del intervalo. En este caso debemos considerar la extension de ϕ dada por(1.11). Tambien debemos admitir la posibilidad de que las integrales tomen valores en [−∞,∞]. Para ellodefinimos ∫

Ω

g dµ =

∫Ω

g+ dµ−∫

Ω

g− dµ, g : Ω→ [−∞,∞] Σ−medible,

siempre que no lleguemos a una expresion no definida, que sucede unicamente cuando∫

Ωg+ dµ =∫

Ωg− dµ = ∞. En el enunciado admitimos implıcitamente que las integrales involucradas tienen sen-

tido.

Demostracion. En caso de que exista a ≤ m ≤ b tal que f = m µ-c.t.p. entonces∫

Ωf dµ = m,∫

Ωϕ f dµ = ϕ(m), y se da la igualdad en (1.12). Llamemos

x =

∫Ω

f dµ.

Si x = a entonces f = a > −∞ µ-c.t.p. mientras que si x = b < ∞ entonces f = b µ-c.t.p. En amboscasos, estamos en la situacion trivial. Por tanto, debemos distinguir tres casos:

Caso 1. Supongamos que a < x < b.Sea β = ϕ′(x−) definida como en (1.4). Gracias a (1.6) tenemos para a < t < x < s < b,

ϕ(x)− ϕ(t)

x− t≤ β ≤ ϕ(s)− ϕ(x)

s− x.

Quitando denominadores en cada desigualdad deducimos que

ϕ(y) ≥ ϕ(x) + (y − x)β (1.13)

para todo y ∈ (a, b).Por tanto:

ϕ(f(ω))− ϕ(x)− (f(ω)− t)β ≥ 0 (1.14)

Page 24: Desigualdades matemáticas clásicas · 2018-02-27 · 3 Resumen En esta memoria recogemos algunas de las desigualdades m as importantes en Matem aticas, sus or ge-nes y aplicaciones.

20 CAPITULO 1. LA DESIGUALDAD DE JENSEN

para todo ω ∈ Ω.Como ϕ es continua, ϕ f es medible. Integrando esta desigualdad, teniendo en cuenta tanto que µ

es una medida de probabilidad como la linealidad y monotonıa de la integral,∫Ω

ϕ(f(ω)) dµ(ω)− ϕ(x)−(∫

Ω

f(ω) dµ(ω)− t)β ≥ 0.

Como∫

Ωf(ω) dµ(ω) = t nos queda ∫

Ω

ϕ(f(ω)) dµ(ω)− ϕ(x) ≥ 0.

Es decir, ∫Ω

ϕ(f(ω)) dµ(ω) ≥ ϕ(x),

que es la desigualdad perseguida.Supongamos, dentro de este primer caso, que ϕ es estrictamente convexa. Entonces (1.13) es una

desigualdad estricta excepto cuando y = x. Por tanto (1.14) es una desigualdad estricta excepto cuandof(ω) = x. Integrando, obtenemos una desigualdad estricta salvo cuando f = x µ-c.t.p.

Caso 2. Sea x = b =∞.Si la funcion ϕ es decreciente se obtiene

ϕ(∞) ≤ ϕ(f),

e integrando llegamos a la desigualdad requerida. Ademas cuando ϕ es estrictamente convexa se tiene

ϕ(∞) < ϕ(f),

e integrando obtenemos una desigualdad estricta, excepto quizas cuando ϕ(∞) = −∞.Si la funcion no es decreciente, tomamos a < c < d <∞ tales que ϕ(c) < ϕ(d). Se tiene que

ϕ(d)− ϕ(c)

d− c≤ ϕ(t)− ϕ(c)

t− c, c < t.

Por tanto,ϕ(d)− ϕ(c)

d− c(t− c) + ϕ(c) ≤ ϕ(t), c < t.

De aquı se deduce que ϕ(∞) = lımt→∞ ϕ(t) =∞. Denotamos

A = ω : f(ω) > c

y consideramos las funciones

f1 = (f − c)χAf2 = cχA + fχΩ\A.

Se tiene que f = f1 + f2 y que f2 ≤ c. Por tanto∫

Ωf2 dµ ≤ c < ∞ y en consecuencia,

∫Ωf1 dµ = ∞.

Ademas,ϕ(d)− ϕ(c)

d− cf1 + ϕ(c) ≤ ϕ(f)χA.

De lo anterior deducimos que∫

Ωϕ(f)χA dµ =∞. Por tanto, siempre que

∫Ωϕ(f) dµ tenga sentido ha de

ser∫

Ωϕ(f) dµ =∞.

Caso 3. Sea x = a = −∞.Basta aplicar el caso 2 a la funcion medible −f y la funcion convexa ϕ(−·).

Page 25: Desigualdades matemáticas clásicas · 2018-02-27 · 3 Resumen En esta memoria recogemos algunas de las desigualdades m as importantes en Matem aticas, sus or ge-nes y aplicaciones.

1.4. PROBLEMAS 21

Comentario. La demostracion que presentamos de la desigualdad de Jensen no sigue los pasos habitualesen terıa de integracion, que consisten en verificar primero el resultado para funciones simples y despuesextenderlo por aproximacion. Notamos que para f =

∑Ni aiχAi simple, con Ai ∈ Σ dos a dos disjuntos

tales que ∪Ni=1Ai = Ω, tenemos∫Ω

ϕ f dµ =

∫Ω

N∑i=1

ϕ(ai)χAid, µ =N∑i=1

ϕ(ai)µ(Ai),

ϕ

(∫Ω

ϕ f dµ)

= ϕ

(N∑i=1

aiµ(Ai)

),

N∑i=1

µ(Ai) = µ(Ω) = 1.

Por tanto, la desigualdad de Jensen (1.12) para una tal funcion f , es una consecuencia inmediata dela desigualdad de Jensen discreta (1.10). Sin embargo, no es evidente como extender el resultado defunciones simples a funciones medibles. Puesto que no imponemos ni que las funciones sean positivas nique sean integrables, no disponemos para nuestros propositos ni del teorema de la convergencia monotonani del teorema de la convergencia dominada.

1.4. Problemas

Pasemos a ver aplicaciones de esta desigualdad en la resolucion de problemas. Primero trataremos unproblema geometrico y despues demostraremos la conocida desigualdad de Nesbitt.

Problema 1. Dado un triangulo inscrito en una circunferencia, probad que el de mayor area es equilatero.

Solucion. Consideremos el triangulo de la figura:

Observamos que los angulos α, β y γ suman π/2.

El area del triangulo formado por los puntos A, B , C es

R2 sin(α) cos(α)

2.

Teniendo en cuenta la formula del angulo doble ( 2 sin(α) cos(α) = sin(2α), α ∈ R) el doble del arealimitada por los puntos A, B y C es (notamos que la region es simetrica respecto al segmento AC)

R2 sin(α) cos(α)

2· 2 =

R2 sin(2α)

2.

Sabiendo esto, pasamos a calcular el area del triangulo total, que es

Area =R2 sin(2α)

2+R2 sin(2β)

2+R2 sin(2γ)

2

=R2(sin(2α) + sin(2β) + sin(2γ))

2

=3R2

2

(sin(2α) + sin(2β) + sin(2γ)

3

).

Page 26: Desigualdades matemáticas clásicas · 2018-02-27 · 3 Resumen En esta memoria recogemos algunas de las desigualdades m as importantes en Matem aticas, sus or ge-nes y aplicaciones.

22 CAPITULO 1. LA DESIGUALDAD DE JENSEN

Puesto que, como ya hemos justificado, la funcion f(x) = sin(x) es estrictamente concava en el intervalo

(0, π) podemos aplicar la desigualdad de Jensen para obtener

3R2

2

(sin(2α) + sin(2β) + sin(2γ)

3

)≥ 3R2

2sin

(2α+ 2β + 2γ

3

)=

3R2

2sin(π

3

)=

3√

3R2

4.

Observamos que la igualdad se da cuando y solo cuando 2α = 2β = 2γ = π/3; o sea, cuando α = β =

γ = π/6. Es decir, el area es maxima cuando se trata de un triangulo equilatero.

Problema 2 (Desigualdad de Nesbitt). Dados a, b, c > 0, probad que

a

b+ c+

b

a+ c+

c

a+ b≥ 3

2

y determinad en que casos se alcanza la igualdad.

Solucion. Esta desigualdad es homogenea, es decir, dada cierta terna de numeros positivos (a, b, c) quecumple la desigualdad, entonces cualquiera que sea t > 0, la terna (ta, tb, tc) tambien la cumple. Ası quepodemos suponer sin perdida de generalidad que a + b + c = 1. De hecho una vez probada con estarestriccion, tomamos (a, b, c) terna cualquiera de numeros positivos; considerando

t = a+ b+ c, a0 =a

t, b0 =

b

t, c0 =

c

t, (1.15)

se tiene quea0 + b0 + b0 = 1

y obtenemos que (a0, b0, c0) cumple la desigualdad; concluimos que

(a, b, c) = t · (a0, b0, c0)

tambien la cumple.

Sean, pues, a, b, c > 0 tales que a+ b+ c = 1. Notamos que en este caso la desigualdad a probar es

a

1− a+

b

1− b+

c

1− c≥ 3

2,

o, lo que es lo mismo,1

3

(a

1− a+

b

1− b+

c

1− c

)≥ 1

2.

Consideramos la funcion ϕ : (0, 1)→ (0,∞) dada por

ϕ(x) =x

1− x, 0 < x < 1. (1.16)

Notamos que ϕ es estrictamente convexa (lo es la funcion x 7→ ψ(x) := 1/x y, por tanto, tambien lafuncion x 7→ ψ(1− x) = 1/(1− x)).

Aplicando la desigualdad de Jensen tenemos que

1

3

(a

1− a+

b

1− b+

c

1− c

)=

1

3(ϕ(a) + ϕ(b) + ϕ(c)) ≥ ϕ

(a+ b+ c

3

)= ϕ(1/3) =

1

2,

tal y como perseguıamos. Ademas, la igual se alcanza, en este caso en el que hemos considerado larestriccion a + b + c = 1, cuando y solo cuando a = b = c. Volviendo al caso general, si t es como en(1.15), la igualdad se alcanza cuando

a

t=b

t=c

t,

es decir, cuando a = b = c.

Page 27: Desigualdades matemáticas clásicas · 2018-02-27 · 3 Resumen En esta memoria recogemos algunas de las desigualdades m as importantes en Matem aticas, sus or ge-nes y aplicaciones.

1.4. PROBLEMAS 23

Comentario. Quizas la manera mas natural de demostrar que la funcion ϕ definida en (1.16) es convexaes apoyarse en que

ψ(x) =1

x, x > 0

lo es. De hecho, se tienen las siguientes propiedades elementales relativas a la convexidad,

Si f es estrictamente convexa en (0, a) entonces x 7→ f(a− x) es estrictamente convexa en (0, a).

Si f es estrictamente convexa en (a, b) y c ∈ R entonces x 7→ c+ f(x) es estrictamente convexa en(a, b).

Por tanto, basta considerar que ϕ(x) = 1 + ψ(1 − x). Por otra parte, con el proposito de demostrar ladesigualdad de Nesbitt de la manera mas elemental posible, incluimos una demostracion de la convexidadde ψ que no usa calculo diferencial. Antes, enunciamos y demostramos una desigualdad elemental.

Lema 2. Para todo x > 0 se tiene que

x+1

x≥ 2,

con igualdad si y solo si x = 1.

Demostracion. Basta tener en cuenta que

x+1

x− 2 =

(√x− 1√

x

)2

≥ 0,

con igualdad si y solo si√x = 1/

√x.

Problema 3. Sean a, b, c numeros positivos tales que a+ b+ c = 1. Probad que:

√a1−ab1−bc1−c ≤ 1

3.

Solucion. Quitando la raız cuadrada (elevando al cuadrado), la desigualdad equivale a

a1−ab1−bc1−c ≤ 1

9.

Tomando logaritmos, llegamos a la desigualdad equivalente

(1− a) log a+ (1− b) log b+ (1− c) log c ≤ 2 log

(1

3

).

Tomamos f(x) = (1− x) log x. Por ser f(x) concava, debido por ejemplo, a que

f ′′(x) =−1− x2

x2< 0, x ∈ (0,∞),

podemos aplicar la desigualdad de Jensen. Obtenemos

f(x1) + f(x2) + f(x3) = 3f(x1) + f(x2) + f(x3)

3

≤ 3f

(x1 + x2 + x3

3

)= 3f

(1

3

)= 3

(1− 1

3

)log

1

3= 2 log

1

3.

Page 28: Desigualdades matemáticas clásicas · 2018-02-27 · 3 Resumen En esta memoria recogemos algunas de las desigualdades m as importantes en Matem aticas, sus or ge-nes y aplicaciones.

24 CAPITULO 1. LA DESIGUALDAD DE JENSEN

Comentario. A continuacion vamos a leer una breve biografıa de Jacob Bernoulli, autor de la desigual-dad que trataremos en el proximo problema.

Jakob Bernoulli (Basilea (Suiza), 27 de diciembre de 1654 - 16 de agosto de 1705), mas conocido comoJacob Bernoulli por la traduccion de su nombre al aleman, fue un matematico y cientıfico ruso. Por deseode su padre estudio Filosofıa y Teologıa, aunque aprovecho su estancia en la Universidad para iniciarseen Matematicas, que era su verdadera vocacion.

Permitio el avance de la teorıa de la probabilidad y las aplicaciones del calculo. En su primer artıculosobre series infinitas (1689) presento la Desigualdad de Bernoulli, que demostraremos a continuacion. Seintereso en las propiedades de curvas como la catenaria, la tractriz, la isocrona y la espiral logarıtmica.En 1960 uso por primera vez la palabra integral, aunque Leibniz prefirio el termino summatorius. Con-tribuyo al estudio de la ecuacion que hoy lleva su nombre y describio la llamada lemniscata de Bernoulli.Su obra mas destacable fue Ars Conjectandi, publicada en 1713 con caracter postumo. En ella se inclu-ye el concepto de numeros de Bernoulli y el principio basico de teorıa de probabilidad: el Teorema deBernoulli, conocido en la actualidad como Ley debil de los grandes numeros.

Problema 4 (Desigualdad de Bernoulli). Sea −1 ≤ x <∞ y a ≥ 1. Entonces

(1 + x)a ≥ 1 + ax.

Solucion. Sea φ(x) = (1 + x)a − 1− ax. Debemos probar que φ(x) ≥ 0, a ≥ 1, −1 ≤ x <∞. Puestoque la funcion xa es convexa, entonces (1 + x)a tambien lo es. Ademas, −1− ax es una funcion convexa.De lo anterior concluimos que (1 + x)a − 1− ax es convexa.Se tiene que φ(0) = φ′(0) = 0. Luego

φ(x)− φ(0)

x− 0> φ′(0), x > 0,

lo que implica que φ(x) > 0, si x > 0.De igual modo tenemos que

φ(x)− φ(0)

x− 0< φ′(0), x < 0,

lo que implica que φ(x) > 0, si x < 0.La igualdad se alcanza si y solo si x = 0 o a = 1.

Page 29: Desigualdades matemáticas clásicas · 2018-02-27 · 3 Resumen En esta memoria recogemos algunas de las desigualdades m as importantes en Matem aticas, sus or ge-nes y aplicaciones.

25

Capıtulo 2

La desigualdad de medias

Comenzamos con la desigualdad clasica entre medias de orden positivo. En esta ocasion empezamoscon una formulacion en terminos de integracion abstracta y mas adelante veremos la version discreta.

2.1. Enunciado y demostracion

Teorema 5. Sean (Ω,Σ, µ) espacio de probabilidad, f ≥ 0 medible y 0 < p < q <∞. Entonces:(∫Ω

fp dµ

)1/p

≤(∫

Ω

fq dµ

)1/q

(2.1)

para toda funcion medible positiva f . Ademas la igualdad se da si y solo si o bien∫

Ωfp dµ = ∞ o bien

f es constante µ-c.t.p.

Demostracion. Notamos que la funcion ϕ : [0,∞)→ [0,∞) dada por ϕ(t) = tq/p es estrictamente convexa.Ademas ϕ(∞) =∞. Por la desigualdad de Jensen (ver Teorema 4),(∫

Ω

fp dµ

)q/p≤∫

Ω

(fp)q/p dµ =

∫Ω

fq dµ.

Donde hemos usado que

ϕ

(∫Ω

fp dµ

)≤∫

Ω

ϕ(fp) dµ.

Elevando a 1/q ambas partes de la desigualdad anterior,(∫Ω

fp dµ

)q/pq=

(∫Ω

fp dµ

)1/p

≤(∫

Ω

fq dµ

)1/q

.

Finalmente, notamos que de todos los casos en los que la desigualdad de Jensen produce igualdad, solopueden darse los dos citados.

Es posible, ademas, considerar medias de orden nulo y negativo y extender los resultados en esteambito. Primero realizamos una extension de la desigualdad de medias a p ∈ [0,∞). Para ello utilizaremosque la funcion logaritmo es estrictamente concava. Notamos que este resultado es equivalente a que lafuncion exponencial sea estrictamente convexa. De hecho se tiene la siguiente propiedad, sencilla decomprobar a partir de la definicion.

Si f es estrictamente convexa y biyectiva entonces su funcion inversa es estrictamente concava.

Teorema 6. Sea (X,Σ, µ) un espacio de probabilidad. Sea p> 0. Se tiene que

exp

(∫Ω

log f dµ

)≤(∫

Ω

fp dµ

)1/p

para toda funcion medible positiva f . Ademas, la igualdad se da si y solo si o bien f es constante µ-c.t.p.o bien

∫Ω

log f dµ =∞.

Page 30: Desigualdades matemáticas clásicas · 2018-02-27 · 3 Resumen En esta memoria recogemos algunas de las desigualdades m as importantes en Matem aticas, sus or ge-nes y aplicaciones.

26 CAPITULO 2. LA DESIGUALDAD DE MEDIAS

Demostracion. Notamos que la funcion log : (0,∞) → (−∞,∞) es funcion concava. Aplicando Jensenllegamos a ∫

Ω

log f dµ ≤ log

(∫Ω

f dµ

). (2.2)

Dado p> 0 cualquiera, aplicando (2.2) a fp,∫Ω

log f dµ =1

p

∫Ω

log fp dµ ≤ 1

plog

(∫Ω

fp dµ

)= log

(∫Ω

fp dµ

)1/p

. (2.3)

Aplicando la funcion exponencial obtenemos la desigualdad requerida. La igualdad se da si y solo si hayigualdad en (2.3). Y segun el teorema 4, eso sucede en dos casos: cuando f es constante µ-c.t.p. y cuando∫

Ωlog f dµ =∞.

Finalmente, damos una extension de la desigualdad de medias a toda la recta real.

Teorema 7. Sea (X,Σ, µ) un espacio de probabilidad. Sea f una funcion medible positiva. Definimos

Mp =

(∫Ω

fp dµ

)1/p

, p 6= 0;

M0 = exp

(∫Ω

log f dµ

).

Se tiene que Mp crece con p. Ademas, si p0 es tal que 0 < Mp0 <∞ y f no es constante µ-c.t.p., entoncesMp0 < Mp para todo p > p0.

Demostracion. Basta demostrar que es creciente en [0,∞) y en (−∞, 0]. El crecimiento en [0,∞) esconsecuencia de los dos teoremas anteriores (Teorema 5 y Teorema 6). Para probar el crecimiento en(−∞, 0] basta probar que Ms ≤Mt si s < t < 0 (caso 1) y que Ms ≤M0 si s < 0 (caso 2).

Caso 1. Considerando lo anterior (Teorema 5) para 1/f obtenemos(∫Ω

(1

f

)pdµ

)1/p

≤(∫

Ω

(1

f

)qdµ

)1/q

, 0 < p < q.

Por tanto, (∫Ω

f−q)−1/q

dµ ≤(∫

Ω

f−p dµ

)−1/p

, 0 < p < q.

Hemos obtenido M−q ≤ M−p para 0 < p < q. Dados s < t < 0, como 0 < −t < −s, se tieneMs ≤Mt, con desigualdad estricta en los casos citados en el enunciado.

Caso 2. Como en el caso anterior, obtenemos

exp

∫Ω

log1

fdµ ≤

(∫Ω

(1

f

)pdµ

)1/p

, p > 0.

Que conduce a

exp

∫Ω

log f dµ ≥(∫

Ω

f−p dµ

)−1/p

, p > 0.

Es decir, obtenemos M−p ≤M0 para p > 0. Equivalentemente, tenemos Ms ≤M0, para s < 0, condesigualdad estricta en los casos citados en el enunciado.

Combinando, obtenemos que la funcion crece en toda la recta real.

Comentario. A la vista del Teorema 7 es natural preguntarse por el valor de los lımites

lımp→±∞

Mp.

En sencillo darse cuenta de que

inf esf(ω) : ω ∈ Ω ≤Mp ≤ sup esf(ω) : ω ∈ Ω, p ∈ R.

Page 31: Desigualdades matemáticas clásicas · 2018-02-27 · 3 Resumen En esta memoria recogemos algunas de las desigualdades m as importantes en Matem aticas, sus or ge-nes y aplicaciones.

2.1. ENUNCIADO Y DEMOSTRACION 27

Por tanto,

inf esf(ω) : ω ∈ Ω ≤ lımp→−∞

Mp,

lımp→∞

Mp ≤ sup esf(ω) : ω ∈ Ω.

Puede demostrarse que, en realidad se da la igualdad. O sea,

lımp→−∞

Mp = inf esf(ω) : ω ∈ Ω,

lımp→∞

Mp = sup esf(ω) : ω ∈ Ω.

Aquı inf es y sup es denotan, respectivamente, el ınfimo y supremo esenciales, o sea, hablando de manerainformal, el ınfimo y supremo despreciando conjuntos de medida nula.

La version discreta de estas desigualdades de medias puede obtenerse a partir del resultado general.Las enuciamos y demostramos como un corolario.

Teorema 8. Sean ai > 0, pi > 0, (i = 1,. . .n) tales que∑ni=1 pi = 1. Definimos

Mp = (p1ap1 + · · ·+ pia

pii + · · ·+ pna

pn)

1/p, p 6= 0;

M0 = ap11 · · · apii · · · a

pnn .

Se tiene que la funcion p 7→Mp es creciente. Ademas, en caso de que la n-tupla (ai)ni=1 no sea constante,

la funcion es estrictamente creciente.

Demostracion. Basta aplicar el Teorema 7 a la medida de probabilidad sobre el conjunto 1, . . . , i, . . . , n,que al conjunto unipuntual i le asigna el valor pi (i = 1, . . . , n). Puesto que nuestras hipotesis conllevanque Mp es siempre finito, se obtiene la desigualdad escricta excepto cuando la n-tupla (ai)

ni=1 es constante

c.t.p. Como el unico conjunto de medida nula es el conjunto vacıo, esto equivale a que la n-tupla seaconstante. Remarcamos como se obtiene la expresion para la media de orden cero en este caso concreto:

M0 = exp

(n∑i=1

log(ai)pi

)= exp

(n∑i=1

log(apii )

)=

n∏i=1

apii .

La version mas clasica del Teorema 8 sucede cuando todos los pi coinciden (y por tanto son iguales a1/n). Si pensamos que pi es un peso que colocamos en cada ındice, esto significa que todos los pi tienenel mismo valor. De modo que el siguiente resultado puede llamarse desigualdad entre medias discreta yequilibrada.

Corolario 1. Sean ai > 0, (i = 1,. . .n). Definimos

Mp = Mp =

(ap1 + · · ·+ api + · · ·+ apn

n

)1/p

, p 6= 0;

M0 = n√a1 · · · ai · · · an.

Se tiene que la funcion p 7→Mp es creciente. Ademas, en caso de que la n-tupla (ai)ni=1 no sea constante

la funcion es estrictamente creciente.

Demostracion. Es una consecuencia inmediata del Teorema 8.

Remarcamos que algunas de las medias involucradas en el Corolario 1 son tan clasicas en la literaturamatematica que tienen su propio nombre.

M1 =a1 + · · ·+ ai + · · ·+ an

nes la media aritmetica.

M2 =

(a2

1 + · · ·+ · · ·+ a2i + · · ·+ a2

n

n

)1/2

es la media cuadratica.

Page 32: Desigualdades matemáticas clásicas · 2018-02-27 · 3 Resumen En esta memoria recogemos algunas de las desigualdades m as importantes en Matem aticas, sus or ge-nes y aplicaciones.

28 CAPITULO 2. LA DESIGUALDAD DE MEDIAS

M−1 =n

1a1

+ · · ·+ 1ai

+ · · ·+ 1an

es la media armonica.

M0 = (a1 · · · ai · · · an)1/n

es la media geometrica.

La relacion entre todas ellas que se obtiene a partir del Corolario 1 es

mına1, . . . , an ≤M−1 ≤M0 ≤M1 ≤M2 ≤ maxa1, . . . , an.

A continuacion, mostramos una demostracion de la desigualdad discreta entre medias (Teorema 8)que no usa herremientas de integracion. Antes, necesitamos tres resultados previos.

Lema 3. Sean 0 < t ≤ 1, 1 < r, 1 ≤ u. Se tiene que

(1 + t(u− 1))r ≤ 1 + t(ur − 1).

Con desigualdad estricta salvo que u = 1.

Demostracion. Para cada r, t fijos consideramos la funcion g : [1,∞)→ R dada por

g(u) = 1 + t(ur − 1)− (1 + t(u− 1))r.

Puesto que g(1) = 0, debemos probar que g(u) > 0 para todo u > 1. Basta probar que g es estrictamentecreciente. Derivando respecto de u ∈ (1,∞),

g′(u) = rtur−1 − rt(1 + t(u− 1))r−1

= rt(ur−1 − (1 + t(u− 1))r−1.

Teniendo en cuenta que 1+ t(u−1) ≤ 1+(u−1) < u, que r−1 > 0 y que tr > 0 obtenemos g(u) ≥ 0.

Lema 4. Sean 0 < t < 1, 0 < p < q <∞, 0 ≤ x, y. Se tiene

((1− t)xp + typ)1/p ≤ ((1− t)xq + tyq)1/q,

con desigualdad estricta excepto cuando x = y.

Demostracion. Denotamos xp = a, yp = b, q/r = r. La desigualdad perseguida se obtiene elevando a lapotencia 1/q la desigualdad

((1− t)a+ tb)r ≤ (1− t)ar + tbr.

Por tanto, basta probar esta otra. Llamamos b/a = u (el caso a = 0 es sencillo). Esta nueva desigualdadse obtiene multiplicando por br la desigualdad

((1− t) + tu)r ≤ (1− t) + tur.

Por tanto, basta probar esta ultima, que puede escribirse de la forma

(1 + t(u− 1))r ≤ 1 + t(ur − 1),

y sabemos que es cierta por el Lema 2.1. La igualdad se da cuando u = 1, o sea x = y.

El siguiente lema esta ıntimamente relacionado con la convexidad de la funcion exponencial.

Lema 5. Sean 0 < t < 1, 0 ≤ x, y. Se tiene

x1−tyt ≤ (1− t)x+ ty,

y la igualdad se da solo cuando x = y.

Demostracion. Puesto que la funcion exponencial es estrictamente convexa, tenemos que

x1−tyt = e(1−t) log x+t log y ≤ (1− t)elog x + telog y.

La igualdad se alcanza si y solo silog x = log y,

es decir, si y solo si x = y.

Page 33: Desigualdades matemáticas clásicas · 2018-02-27 · 3 Resumen En esta memoria recogemos algunas de las desigualdades m as importantes en Matem aticas, sus or ge-nes y aplicaciones.

2.1. ENUNCIADO Y DEMOSTRACION 29

Pasamos ahora a demostrar la prometida demostracion elemental de la version de la desigualdad deJensen que da el Teorema 8 . Utilizaremos el metodo de induccion.

Demostracion elemental del Teorema 8. Para 0 < p < q debemos demostrar que

(λ1ap1 + · · ·+ λna

pn)1/p ≤ (λ1a

q1 + · · ·+ λna

qn)1/q

con λ1 + · · ·+ λn = 1. Acometemos la demostracion por induccion.• Base de induccion. Para n = 2 la desigualdad del enunciado es

(λ1ap1 + λ2a

p2)1/p ≤ (λ1a

q1 + λ2a

q2)1/q (2.4)

con λ1 + λ2 = 1.

Llamando (1− t) = λ1, t = λ2, x = a1 y y = a2, esta desigualdad es cierta por el Lema 4.

• Paso de induccion. Supongamos que la desigualdad (2.4) es cierta para un numero natural n ≥ 2 yprobemos que, en tal caso, tambien es cierta para n+ 1. O sea, perseguimos

(λ1ap1 + · · ·+ λna

pn + λn+1a

pn+1)1/p ≤ (λ1a

q1 + · · ·+ λna

qn + λn+1a

qn+1)1/q

con λ1 + · · ·+ λn + λn+1 = 1.Operando,

(λ1ap1 + · · ·+ λna

pn + λn+1a

pn+1)1/p = ((1− λn+1)Ap + λn+1a

pn+1)1/p

donde

A =

(λ1

1− λn+1ap1 + · · ·+ λn

1− λn+1apn

)1/p

y se cumpleλ1

1− λn+1+ · · ·+ λn

1− λn+1= 1.

Aplicando tanto la hipotesis de induccion como que la base de induccion es cierta,

((1− λn+1)Apλn+1apn+1)1/p ≤ ((1− λn+1)Aq + λn+1a

qn+1)1/q

((1− λn+1)

(λ1

1− λn+1aq1 + · · ·+ λn

1− λn+1aqn

)q/q+ λn+1a

qn+1

)1/q

=(λ1a

q1 + · · ·+ λna

qn + λn+1a

qn+1

)1/q.

Probar que M0 ≤Mp si 0 ≤ p, consiste en ver que

aλ01 · · · aλnn ≤ (λ1a

p1 + · · ·+ λna

pn)1/p.

Con la transformacion api = bi basta ver que

bλ11 · · · bλnn ≤ λ1b1 + · · ·+ λnbn.

Demostramos esta desigualdad por induccion. La base de induccion n = 2 es una consecuencia directadel Lema 5. Supongamos que es cierta para un cierto natural n. Dados bi ≥ 0, λi ≥ 0 con

∑n+1i=1 λi = 1,

aplicando como antes la base de induccion y la hipotesis de induccion,

bλ11 · · · bλnn b

λn+1

n+1 =

(b

λ11−λn+1

1 · · · bλn

1−λn+1n

)1−λn+1

bλn+1

n+1

≤ (1− λn+1)bλ1

1−λn+1

1 · · · bλn

1−λn+1n + λn+1bn+1

≤ (1− λn+1)

(λ1

1− λn+1b1 + · · ·+ λn

1− λn+1bn

)+ λn+1bn+1

= λ1b1 + · · ·+ λnbn + λn+1bn+1,

como querıamos probar.El crecimiento en el intervalo (−∞, 0] se demuestra, como de costumbre, utilizando el resultado en

[0,∞) a la n-tupla (1/ai)ni=1. El crecimiento estricto se debe a que la igualdad en el Lema 4 y en el Lema 5

es estricta excepto cuando x = y.

Page 34: Desigualdades matemáticas clásicas · 2018-02-27 · 3 Resumen En esta memoria recogemos algunas de las desigualdades m as importantes en Matem aticas, sus or ge-nes y aplicaciones.

30 CAPITULO 2. LA DESIGUALDAD DE MEDIAS

2.2. Problemas

Pasemos a ver aplicaciones de esta desigualdad a la resolucion de problemas.

Problema 5. Demostrar mediante las desigualdades de medias el Lema 2.

Solucion. Sea x > 0. aplicando la desigualdad entre la media aritmetica y la geometrica,

x+1

x= 2 · 1

2

(x+

1

x

)=

(x · 1

x

)1/2

= 1.

Presentamos ahora una nueva prueba de la desigualdad de Bernoulli (Problema 5), que ya resolvimosen la segunda seccion basandonos en la convexidad de funciones.

Problema 6 (Desigualdad de Bernoulli). Sea a ≥ 1 y −1 ≤ x <∞. Entonces

(1 + x)a ≥ 1 + ax.

Solucion. Haciendo el cambio t = ax y utilizando la desigualdad entre medias (ponderadas) de orden 0y 1, con pesos 1 y 1

n , nos queda

(1 +1

a· t) ≥ (1 + t)1/a, t > −1.

Por tanto, el resultado se sigue de la desigualdad entre la media aritmetica y geometrica.

Problema 7. Sean a, b, c numeros positivos. Probad que:

3(a+ b+ c) ≥ 83√abc+

3

√a3 + b3 + c3

3.

Solucion. Sea Mk la media de orden k de a, b, c. La desigualdad del enunciado equivale a tener

9M1 ≥ 8M0 +M3,

que, a su vez, equivale a

M1 ≥8

9M0 +

1

9M3.

Remarcamos los valores de M0, M1, M3.

M0 =3√abc,

M1 =a+ b+ c

3,

M3 =3

√a3 + b3 + c3

3.

Utilizando la desigualdad entre medias (ponderadas) de orden 1 y orden 3 tenemos que

8

9M0 +

1

9M3 ≤

(8

9M3

0 +1

9M3

3

)1/3

.

Por tanto, basta probar que

M1 ≥(

8

9M3

0 +1

9M3

3

)1/3

,

o, lo que es lo mismo,

M31 ≥

(8

9M3

0 +1

9M3

3

)1/3

.

Page 35: Desigualdades matemáticas clásicas · 2018-02-27 · 3 Resumen En esta memoria recogemos algunas de las desigualdades m as importantes en Matem aticas, sus or ge-nes y aplicaciones.

2.2. PROBLEMAS 31

Volviendo a la expresion en a, b, c esta desigualdad equivale a(a+ b+ c

3

)3

≥ 8

9abc+

1

9

a3 + b3 + c3

3.

Operando, obtenemos la siguiente cadena de desigualdades equivalentes:

a3 + b3 + c3 + 3(ab2 + ac2 + a2b+ a2c+ c2b+ b2c+ 2abc)

33≥ 8

9abc+

1

9

a3 + b3 + c3

3.

ab2 + ac2 + a2b+ a2c+ c2b+ b2c+ 2abc ≥ 8abc.

ab2 + ac2 + a2b+ a2c+ c2b+ b2c ≥ 6abc.

El termino de la derecha es una media aritmetica, cuya correspondiente media geometrica es

6√a2bac2a2ba2cc2bcb2c = abc.

Por tanto, el resultado se sigue de la desigualdad entre la media geometrica y aritmetica.

Problema 8. Sean a, b, c numeros reales positivos tales que abc = 1. Probad que

1

a3(b+ c)+

1

b3(c+ a)+

1

c3(a+ b)≥ 3

2.

Solucion. Hacemos el cambio x = 1a , y = 1

b y z = 1c para intentar quitar terminos del denominador.

Sustituimos en el primer sumando y operamos

1

a3(b+ c)=

1a2(b+c)bc

=bc

a2(b+ c)=

1a2

c+bbc

=x2

y + z.

Hacemos lo mismo en el segundo sumando

1

b3(c+ a)=

1b2(c+a)ca

=ca

b2(c+ a)=

1b2

c+aca

=y2

x+ z.

Y por ultimo, sustituimos en el tercero

1

c3(a+ b)=

1c2(a+b)ab

=ab

c2(a+ b)=

1c2

a+bab

=z2

x+ y.

Agrupamos todo y llegamos a que la desigualdad inicial es equivalente a tener

1

a3(b+ c)+

1

b3(c+ a)+

1

c3(a+ b)=

x2

y + z+

y2

x+ z+

z2

x+ y≥ 3

2.

Consideremos ahora la funcion f(x) = x−1, para x > 0. Hemos probado antes que esta funcion es convexa(empleando calculo diferencial, podemos demostrarlo viendo que la segunda derivada es positiva, de hechof ′′(x) = 2/x3 > 0.)Aplicando la desigualdad de Jensen,

x2

y + z+

y2

x+ z+

z2

x+ y

= xf

(y + z

x

)+ yf

(z + x

y

)+ zf

(x+ y

z

)≥ (x+ y + z)f

((y + z) + (z + x) + (x+ y)

x+ y + z

)= (x+ y + z)f

(2(x+ y + z)

x+ y + z

)= (x+ y + z)f(2)

=x+ y + z

2.

Page 36: Desigualdades matemáticas clásicas · 2018-02-27 · 3 Resumen En esta memoria recogemos algunas de las desigualdades m as importantes en Matem aticas, sus or ge-nes y aplicaciones.

32 CAPITULO 2. LA DESIGUALDAD DE MEDIAS

Como x+ y + z ≥ 3 3√xyz = 3, nos queda que

x2

y + z+

y2

x+ z+

z2

x+ y≥ x+ y + z

2=

3

2.

Problema 9. Sean a, b, c numeros positivos tales que a+ b+ c = 1. Probad que

√a1−ab1−bc1−c ≤ 1

3.

Solucion. Puesto que

1− a2

+1− b

2+

1− c2

=3− (a+ b+ c)

2=

3− 1

2= 1

podemos utilizar la desigualdad entre las medias armonica y aritmetrica ponderadas con estos pesos.Obtenemos

√a1−ab1−bc1−c = a(1−a)/2b(1−b)/2c(1−c)/2 ≤

(1− a

2

)a+

(1− b

2

)b+

(1− c

2

).

Por tanto, es suficiente probar que(1− a

2

)a+

(1− b

2

)b+

(1− c

2

)≤ 1

3.

Operando, obtenemos la siguiente cadena de desigualdades equivalentes a la de arriba:(1− a

2

)a+

(1− b

2

)b+

(1− c

2

)c ≤ 1

3.

1

2[a+ b+ c− (a2 + b2 + c2)] ≤ 1

3.

1

2[1− (a2 + b2 + c2)] ≥ 1

3.

a2 + b2 + c2 ≥ 1

3.

a2 + b2 + c2

3≥ 1

9.√

a2 + b2 + c2

3≥ 1

3.

El termino de la izquierda es una media cuadratica cuya correspondiente media aritmetica es

a+ b+ c

3=

1

3.

Por tanto, el resultado se sigue de la desigualdad entre las medias aritmetica y geometrica.

A continuacion, proponemos una demostracion alternativa de la desigualdad de Nesbitt (ver Proble-ma 2).

Problema 10 (Desigualdad de Nesbitt). Sean a, b, c numeros positivos. Probad que:

a

b+ c+

b

c+ a+

c

a+ b≥ 3

2

y que la igualdad se da solamente cuando a=b=c.

Page 37: Desigualdades matemáticas clásicas · 2018-02-27 · 3 Resumen En esta memoria recogemos algunas de las desigualdades m as importantes en Matem aticas, sus or ge-nes y aplicaciones.

2.2. PROBLEMAS 33

Solucion. Esta desigualdad es homogenea, es decir, dada cierta terna de numeros positivos (a, b, c) quecumple la desigualdad, entonces cualquiera que sea t > 0, la terna (ta, tb, tc) tambien la cumple. Ası quepodemos suponer sin perdida de generalidad que a+ b+ c = 1. Sean: x = 1− a, y = 1− b, z = 1− c. Setiene que

a

b+ c+

b

c+ a+

c

a+ b=

1

b+ c− b+ c

b+ c+

1

c+ a− c+ a

c+ a+

1

a+ b− a+ b

a+ b

=1

b+ c+

1

c+ a+

1

a+ b

=1

1− a+

1

1− b+

1

1− c

=1

x+

1

y+

1

z.

Por tanto, tenemos que probar que1

x+

1

y+

1

z≥ 9

2,

donde la terna de numeros x, y, z cumple con x, y, z > 0, x+ y + z = 3− 1 = 2.

Ahora nuestra desigualdad objetivo es equivalente a

11x + 1

y + 1z

≤ 2

9,

que a su vez equivale a3

1x + 1

y + 1z

≤ 2

3.

La parte izquierda de la desigualdad es la media armonica de x, y, z. La media aritmetica de estos tresnumeros es

x+ y + z

3=

2

3.

Como sabemos que la media armonica es menor que la aritmetica, queda probada la desigualdad.

Problema 11. [Olimpiada Matematica Espanola, 2009]

Sean a, b, c numeros positivos tales que abc = 1. Probad que(a

1 + ab

)2

+

(b

1 + bc

)2

+

(c

1 + ca

)2

≥ 3

4.

Solucion. Esta desigualdad es equivalente a(a

1+ab

)2

+(

b1+bc

)2

+(

c1+ca

)2

3≤ 1

4

y elevando ambos miembros a 1/2 llegamos a√√√√( a1+ab

)2

+(

b1+bc

)2

+(

c1+ca

)2

3≤ 1

2.

Nos damos cuenta de que la parte izquierda corresponde con la media cuadratica de los terminos(a

1 + ab

)2

,

(b

1 + bc

)2

,

(c

1 + ca

)2

,

es decir

M2 =

√√√√( a1+ab

)2

+(

b1+bc

)2

+(

c1+ca

)2

3.

Page 38: Desigualdades matemáticas clásicas · 2018-02-27 · 3 Resumen En esta memoria recogemos algunas de las desigualdades m as importantes en Matem aticas, sus or ge-nes y aplicaciones.

34 CAPITULO 2. LA DESIGUALDAD DE MEDIAS

Queremos ver que M2 ≥ 12 , pero como sabemos que M2 ≥M1, basta con probar que M1 ≥ 1

2 . Veamos sise cumple.

M1 =

(a

1+ab

)+(

b1+bc

)+(

c1+ca

)3

≥ 1

2(a

1 + ab

)+

(b

1 + bc

)+

(c

1 + ca

)≥ 3

2.

Hacemos el cambio: a = y/x, b = z/y, c = x/z, sustituimos en la expresion anterior y operamos.(yx

1 + yxzy

)+

(zy

1 + zyxz

)+

( xz

1 + xzyx

)≥ 3

2( yx

1 + zx

)+

(zy

1 + xy

)+

( xz

1 + yz

)≥ 3

2( yxx+zx

)+

(zy

y+xy

)+

(xzz+yz

)≥ 3

2.

Y llegamos a (y

x+ z

)+

(z

y + x

)+

(x

z + y

)≥ 3

2,

que sabemos que es cierta ya que se trata de la desigualdad de Nesbitt, probada en el problema anterior.

Page 39: Desigualdades matemáticas clásicas · 2018-02-27 · 3 Resumen En esta memoria recogemos algunas de las desigualdades m as importantes en Matem aticas, sus or ge-nes y aplicaciones.

35

Capıtulo 3

La desigualdad de Cauchy-Schwarz

La desigualdad para sumas fue publicada por Augustin Louis Cauchy en 1821, mientras que la co-rrespondiente desigualdad para integrales fue establecida por Viktor Yakovlevich Bunyakovsky (1859) yredescubierta por Hermann Amandus Schwarz en 1888. Quizas lo mas correcto (aunque no habitual) esllamarla desigualdad de Cauchy-Bunyakovsky-Schwarz.

3.1. Augustin Louis Cauchy

Augustin Louis Cauchy (Parıs, 21 de agosto de 1789 – Sceaux, 23 de mayo de 1857) fue un matematicoe ingeniero frances. Estudio Ingenierıa en la Escuela Politecnica de Parıs, donde trabajo como profesorde mecanica (1816), y posteriormente, fue ascendido a miembro de la Academia Francesa de las Ciencias.

Su contribucion matematica es especialmente notable en la rama del analisis matematico. En 1814 pu-blico la memoria de la integral definida, que llego a ser la base de la teorıa de las funciones complejas.Preciso los conceptos de funcion, lımite y continuidad de manera muy similar a como los conocemos enla actualidad, tomando el concepto de lımite como punto de partida del analisis y la nocion de corres-pondencia como idea de funcion.

Tambien hizo aportaciones a la probabilidad, las ecuaciones diferenciales y la fısica matematica.Investigo la convergencia y divergencia de las series infinitas y demostro el Teorema del numero poligonalde Fermat, al que se habıan dedicado ilustres matematicos contemporaneos sin exito.Poco antes de morir pronuncio la frase: “No me imagino una vida mas plena que una vida dedicada ala matematica”, mostrando arrepentimiento por el que consideraba su unico error en la vida: no haberdedicado mas tiempo a las matematicas.

Page 40: Desigualdades matemáticas clásicas · 2018-02-27 · 3 Resumen En esta memoria recogemos algunas de las desigualdades m as importantes en Matem aticas, sus or ge-nes y aplicaciones.

36 CAPITULO 3. LA DESIGUALDAD DE CAUCHY-SCHWARZ

3.2. Karl Hermann Schwarz

Karl Hermann Amandus Schwarz (Polonia, 25 de enero de 1843 – Berlın, 30 de noviembre de 1921),fue un matematico aleman conocido por su trabajo en analisis matematico. Estudio Quımica en Berlınaunque fue persuadido por Kummer y Weierstrass para que se centrara en las matematicas.

Desde 1875 trabajo en la Universidad de Gotinga, donde trato temas como la teorıa de funciones,geometrıa diferencial y calculo de variaciones. Su trabajo Busqueda de una superficie mınima, acabadoen 1867, fue incluido en su Coleccion de artıculos matematicos(1890).En 1892 se convirtio en miembro de la Academia de las Ciencias de Berlın y fue profesor de la Universidadde esta ciudad. Entre sus alumnos mas importantes se encuentran Lipot Fejer, Paul Koebe y LeonLichtenstein.

3.3. Viktor Yakovlevich Bunyakovsky

Viktor Yakovlevich Bunyakovsky (Ucrania, 16 de diciembre - San Petersburgo, 12 de diciembre de1889) fue un matematico ruso, miembro y posterior vicepresidente de la Academia de Ciencias de SanPetersburgo.Estudio Matematicas en la Universidad de la Sorbona, donde se doctoro bajo la supervision de Cauchy.Dedico gran parte de su vida a la ensenanza e investigacion. Contribuyo de manera significativa en lateorıa de numeros y en la teorıa de la probabilidad, aunque tambien hizo aportaciones en ciencias comola fısica, mecanica y finanzas. Se le atribuyen mas de 150 publicaciones sobre trabajos de investigacion.

Page 41: Desigualdades matemáticas clásicas · 2018-02-27 · 3 Resumen En esta memoria recogemos algunas de las desigualdades m as importantes en Matem aticas, sus or ge-nes y aplicaciones.

3.4. INTRODUCCION 37

Es conocido por el descubrimiento de la desigualdad de Cauchy-Schwarz, ya que la probo para el casode dimension infinita en 1859, varios anos antes de que lo hiciera Hermann Schwarz, pese a no obtenernunca el reconocimiento merecido. En el ano 1875 la Academia de Ciencias de San Petersburgo creo unpremio que lleva su nombre como homenaje a su trayectoria en la investigacion matematica.

3.4. Introduccion

Consideramos el espacio vectorial sobre el cuerpo de los reales, denotado por R, y una aplicacion

E × E → R, (x, y) 7→ 〈x, y〉

que verifica

(a) 〈x, y〉 = 〈y, x〉 para todo x, y ∈ E.

(b) 〈x, x〉 ≥ 0 para todo x ∈ E.

(c) 〈λx+ µy, z〉 = λ〈x, z〉+ µ〈y, z〉 para todo λ, µ ∈ R, para todo x, y ∈ E.

Definimos a partir de esta aplicacion

‖ · ‖ : E → [0,∞), x 7→ ‖x‖ =√〈x, x〉. (3.1)

Remarcamos que no exigimos la condicion

(d) Si 〈x, x〉 = 0 entonces x = 0,

que junto con (a), (b), (c) hace de 〈·, ·〉 un producto escalar. Realizamos el estudio en este ambito algomas general debido a que uno de los ejemplos mas importantes de aplicacion es el espacio L2(µ), definidoa partir de un espacio de medida (Ω,Σ, µ) mediante

L2(µ) = f : Ω→ R : medible,

∫Ω

f2 dµ <∞,

dotado con la aplicacion

L2(µ)× L2(µ)→ R, (f, g) 7→∫

Ω

fg dµ, (3.2)

verifica las condiciones (a), (b), (c) pero no necesariamente la condicion (d). Demostramos estos hechoscon precision. Necesitamos, para ello, un lema previo.

Lema 6. Para todo x, y ∈ R se tiene2xy ≤ x2 + y2,

con igualdad si y solo si x = y.

Demostracion. Basta considerar que

x2 + y2 − 2xy = (x− y)2 ≥ 0.

Demostracion alternativa del Lema 6. La desigualdad equivale a

xy ≤ x2 + y2

2.

Denotamos a = |x|, b = |y|. Usando la desigualdad entre la media geometrica y cuadratica,

xy ≤ |xy| = ab =(√

ab)2

(√a2 + b2

2

)2

=a2 + b2

2=x2 + y2

2,

con igualdad si y solo si |x| = |y| junto con xy = |xy|, que equivalen a x = y.

Page 42: Desigualdades matemáticas clásicas · 2018-02-27 · 3 Resumen En esta memoria recogemos algunas de las desigualdades m as importantes en Matem aticas, sus or ge-nes y aplicaciones.

38 CAPITULO 3. LA DESIGUALDAD DE CAUCHY-SCHWARZ

Teorema 9. Sea (X,Σ, µ) un espacio de medida. Entonces L2(µ) es un espacio vectorial dotado de laaplicacion (3.2) que verifica las condiciones (a), (b), (c).

Demostracion. Teniendo en cuenta las propiedades de la integral, hay dos hechos crıticos a demostrar.Son

(i) Si f , g ∈ L2(µ), entonces f + g ∈ L2(µ). Por tanto L2(µ) es un espacio vectorial.

(ii) Si f , g ∈ L2(µ), entonces fg ∈ L1(µ). Por tanto la aplicacion (3.2) esta bien definida.

Para probar (i) notamos que f + g es medible y que, gracias al Lema 6

(f + g)2 = f2 + g2 + 2fg ≤ 2(f2 + g2).

Por tanto ∫Ω

(f + g)2 dµ ≤ 2

∫Ω

f2 dµ+ 2

∫Ω

g2 dµ <∞.

Para probar (i) notamos que fg es medible y que, tambien gracias al Lema 6

|fg| ≤ f2 + g2

2.

Por tanto ∫Ω

|fg| dµ ≤ 1

2

∫Ω

f2 dµ+1

2

∫Ω

g2 dµ <∞.

Antes de volver al marco general notamos que, para el caso concreto del espacio L2(µ), la aplicaciondada por (3.1), que denotaramos mediante ‖ · ‖2, viene dada por

‖ · ‖2 : L2(µ)→ [0,∞), f 7→ ‖f‖2 =

(∫Ω

f2 dµ

)1/2

.

Ademas,f ∈ L2(µ) : 〈f, f〉 = 0 = f : Ω→ R medible : f = 0 µ-c.t.p..

3.5. La desigualdad de Cauchy-Schwarz

Teorema 10 (Desigualdad de Cauchy-Schwarz). Sean E un espacio vectorial y una aplicacion 〈·, ·〉 : E×E → R que verifican (a), (b), (c). Definimos ‖ · ‖ como en (3.1). Se tiene que

|〈x, y〉| ≤ ‖x‖ ‖y‖

para todo x, y ∈ E. Ademas si se obtiene la igualdad, existe (s, t) 6= (0, 0) tal que

sx+ ty ∈ N := z ∈ E : 〈z, z〉 = 0.

Demostracion. Distinguimos dos casos.

Si 〈y, y〉 = 0. Consideremos los vectores tx + y, con t un numero real cualquiera. Calculamos elproducto 〈tx+ y, x+ y〉 que sera no negativo gracias a la propiedad (b). Tenemos

0 ≤ 〈tx+ y, tx+ y〉 = t2〈x, x〉+ 2t〈x, y〉+ 〈y, y〉 = t2〈x, x〉+ 2t〈x, y〉.

Si aplicamos esta desigualdad con t = ±s, donde s > 0 obtenemos

0 ≤ s2〈x, x〉 ± 2s〈x, y〉.

Multiplicando por 1/(2s) obtenemos

0 ≤ s

2〈x, x〉 ± 〈x, y〉.

Haciendo tender s a cero llegamos a 0 ≤ ±〈x, y〉. Es decir |〈x, y〉| = 0, por lo que se tiene la igualdad.Obviamente, 0 · x+ 1 · y ∈ N .

Page 43: Desigualdades matemáticas clásicas · 2018-02-27 · 3 Resumen En esta memoria recogemos algunas de las desigualdades m as importantes en Matem aticas, sus or ge-nes y aplicaciones.

3.6. LA DESIGUALDAD DE CAUCHY-SCHWARZ EN LA TEORIA DE INTEGRACION 39

Si 〈y, y〉 > 0. Consideremos los vectores x + ty, con t un numero real cualquiera. Calculamos elproducto 〈x+ ty, x+ ty〉 que sera no negativo gracias a la propiedad (b). Tenemos

0 ≤ 〈x+ ty, x+ ty〉 = 〈x, x〉+ t2〈y, y〉+ 2t〈x, y〉.

Si escogemos t = −〈x, y〉〈y, y〉

tenemos:

0 ≤ 〈x, x〉+〈x, y〉2

〈y, y〉− 2〈x, y〉2

〈y, y〉= 〈x, x〉 − 〈x, y〉

2

〈y, y〉,

de donde despejando 〈x, y〉 se obtiene:

〈x, y〉2 ≤ 〈x, x〉〈y, y〉,

luego sacando raıces cuadradas positivas:

|〈x, y〉| ≤ ‖x‖ ‖y‖ .

Ademas, cuando se da la igualdad, 〈x+ ty, x+ ty〉 = 0. Es decir 1 · x+ t · y ∈ N .

Continuamos aplicando la desigualdad de Cauchy-Schwarz en casos concretos.

3.6. La desigualdad de Cauchy-Schwarz en la teorıa de integra-cion

Teorema 11. Sea (Ω,Σ, µ) espacio de medida. Sean f , g medibles tales que

∫f2 dµ,

∫g2 dµ < ∞.

Entoncesfg ∈ L1(µ)

y se tiene que ∣∣∣∣∫ fg dµ

∣∣∣∣ ≤ (∫ f2 dµ

)1/2(∫g2 dµ

)1/2

. (3.3)

Ademas, si se da la igualdad, entonces existe (s, t) 6= (0, 0) tal que sf + tg = 0 µ-c.t.p.

Demostracion. Es una consecuencia directa de combinar el Teorema 9 con el Teorema 10.

3.7. La desigualdad de Cauchy-Schwarz para sumas

Teorema 12. Para todo n ∈ N y todo par de n-tuplas de numeros reales (xi)ni=1, (yi)

ni=1 se tiene que∣∣∣∣∣

n∑i=1

xiyi

∣∣∣∣∣ ≤(

n∑i=1

x2i

)1/2( n∑i=1

y2i

)1/2

.

Ademas, si se da la igualdad, las n-tuplas (xi)ni=1, (yi)

ni=1 son linealmente dependientes.

Demostracion. Basta aplicar el Teorema 11 a la medida de contar sobre el conjunto i ∈ N : i ≤ n. Esdecir, la medida que a cada conjunto unipuntual i le asigna el valor 1, i = 1,. . . , n. Remarcamos que,en este caso la unica funcion nula c.t.p. es la n-tupla nula.

El Teorema 12 es la version mas clasica y elemental de la desigualdad de Cauchy-Schwarz, quepodrıamos llamar version discreta de la desigualdad de Cauchy-Schwarz. Aunque hemos llegado has-ta ella desde un marco muy general (para ser preciso, desde el marco de los espacios de Hilbert) nohemos utilizado en su demostracion conceptos sofisticados ni tecnicas avanzadas. De modo que interpre-tamos que la demostracion que hemos dado del Teorema 12 es elemental. No obstante, presentamos otrademostracion basada en el principio de induccion.

Page 44: Desigualdades matemáticas clásicas · 2018-02-27 · 3 Resumen En esta memoria recogemos algunas de las desigualdades m as importantes en Matem aticas, sus or ge-nes y aplicaciones.

40 CAPITULO 3. LA DESIGUALDAD DE CAUCHY-SCHWARZ

Demostracion alternativa del Teorema 12. Hacemos induccion en n ∈ N. Para n = 1 es obvio, peronecesitamos situar la base de induccion en n = 2.• Base de induccion. Para n = 2 la desigualdad del enunciado es

|x1y1 + x2y2| ≤ (x21 + x2

2)1/2(y21 + y2

2)1/2,

que, elevando al cuadrado, equivale a

(x1y1 + x2y2)2 ≤ (x21 + x2

2)(y21 + y2

2).

Operando obtenemos la cadena de desigualdades equivalentes

x21y

21 + x2

2y22 + 2x1y1x2y2 ≤ x2

1y21 + x2

2y21 + x2

1y22 + x2

2y22 .

2x1y1x2y2 ≤ x22y

21 + x2

1y22 .

Esta ultima se obtiene aplicando el Lema 6 con x = x1y2, y = x1y2. La igualdad se da cuando x1y2−x1y2 =0 son linealmente dependientes. Es decir (x1, x2), (y1, y2) son linealmente dependientes.• Paso de induccion. Supongamos que la desigualdad es cierta para un numero natural n ≥ 2 y

probemos que, en tal caso, tambien es cierta para n + 1. Si existe i tal que xi = yi = 0, el resultadose desprende directamente de la hipotesis de induccion. Suponemos por tanto, que (xi, yi) 6= (0, 0) paratodo i. Por simetrıa entre los vectores, podemos suponer ademas que xn+1 6= 0. Aplicando la hipotesisde induccion,∣∣∣∣∣

n+1∑i=1

xiyi

∣∣∣∣∣ ≤∣∣∣∣∣n∑i=1

xiyi

∣∣∣∣∣+ |xn+1yn+1| ≤

(n∑i=1

x2i

)1/2( n∑i=1

y2i

)1/2

+ |xn+1| |yn+1| .

Aplicando ahora la base de induccion (el caso n = 2) obtenemos

(n∑i=1

x2i

)1/2( n∑i=1

y2i

)1/2

+ |xn+1yn+1| ≤

( n∑i=1

x2i

)2/2

+ x2n+1

1/2( n∑i=1

y2i

)2/2

+ y2n+1

1/2

=

(n+1∑i=1

x2i

)1/2(n+1∑i=1

y2i

)1/2

.

La igualdad se da cuando suceden, simultaneamente,

(xi)ni=1, (yi)

ni=1 son linealmente dependientes.∑n

i=1 xiyi tiene el mismo signo que xn+1yn+1.

(∑n+1i=1 x

2i )

1/2, |xn+1|), (∑n+1i=1 y

2i )1/2, |yn+1|) son linealmente dependientes.

De aquı se deduce que (xi)n+1i=1 , (yi)

n+1i=1 son linealmente dependientes. Por tanto existen s, t ∈ R tal que

yi = sxi, (i = 1, . . . , n), |yn+1| = t|xn+1|,

(n+1∑i=1

y2i

)1/2

= t

(n+1∑i=1

x2i

)1/2

.

De quı se deduce que |s| = t Por tanto |yn+1| = |sxn+1|. Teniendo en cuenta la coincidencia de signos,yn+1 = sxn+1.

3.8. El Lema de Titu

Titu Andreescu (Rumanıa, 1956), profesor en la Universidad de Texas, es conocido por su destacabletrabajo y participacion en olimpiadas internacionales de matematicas. Ha llegado a ser director en lasolimpiadas AMC y MOP, entrenador principal de las IMO estadounidenses y presidente de USAMO. Hapublicado numerosos libros sobre problemas y ejercicios de olimpiadas matematicas, entre los que destacala desigualdad que trataremos a continuacion, conocida como el Lema de Titu.

Page 45: Desigualdades matemáticas clásicas · 2018-02-27 · 3 Resumen En esta memoria recogemos algunas de las desigualdades m as importantes en Matem aticas, sus or ge-nes y aplicaciones.

3.9. LA DESIGUALDAD TRIANGULAR 41

Teorema 13 (Lema de Titu). Sean a1, a2, . . .an, b1, b2, . . . bn numeros positivos. Entonces

a21

b1+a2

2

b2· · ·+ a2

n

bn≥ (a1 + a2 · · ·+ an)2

b1 + b2 + · · ·+ bn

Demostracion. Notemos que la desigualdad del enunciado es equivalente a(a2

1

b1+a2

2

b2+ · · ·+ a2

n

bn

)(b1 + · · · bn) ≥ (a1 + · · · an)2

Hacemos los cambios ai/√bi = ai ,

√bi = bi, con 1 ≤ i ≤ n y llegamos a

(a21 + a2

2 + · · ·+ a2n)(b21 + b22 + · · ·+ b2n) ≥ (a1b1 + a2b2 + · · ·+ anbn)2.

Concluimos la demostracion observando que se trata de la desigualdad de Cauchy-Schwarz para dossecuencias a1, . . . , an, b1, . . . , bn de numeros reales positivos.La igualdad se alcanza si existe una constante λ tal que ai = λ · xi para todo i = 1, . . . , n.

3.9. La desigualdad triangular

La desigualdad de Cauchy-Schwarz nos proporciona un argumento a la vez sencillo y preciso parademostrar que los espacios euclıdeos cumplen la desigualdad triangular. Esto es, si definimos en Rn ladistancia

d(x, y) =

(n∑i=1

(yi − xi)2

)1/2

~x = (x1, . . . , xn), ~y = (y1, . . . , yn),

se tiene qued(~x, ~y) ≤ d(~x, ~z) + d(~z, ~y), ~x, ~y, ~z ∈ Rn. (3.4)

Si denotamos

‖~x‖2 =

(n∑i=1

x2i

)1/2

, ~x = (x1, . . . , xn),

la desigualdad (3.4) se obtiene como consecuencia directa de la desigualdad

‖~x+ ~y‖2 ≤ ‖~x‖2 + ‖~y‖2, ~x, ~y ∈ Rn.

Este hecho es consecuencia inmediata de aplicar el siguiente resultado a la medida de contar sobre elconjunto 1, 2.

Teorema 14. Sean E un espacio vectorial y una aplicacion 〈·, ·〉 : E×E → R que verifican (a), (b), (c).Definimos ‖ · ‖ como en (3.1). Se tiene que

‖x+ y‖ ≤ ‖x‖+ ‖y‖

para todo x, y ∈ E. Ademas, si se obtiene la igualdad existen s, t ≥ 0 con (s, t) 6= (0, 0) tal que

sx+ ty ∈ N := z ∈ E : 〈z, z〉 = 0.

Demostracion. Gracias a la desigualdad de Cauchy-Schwarz,

‖x+ y‖2 = ‖x‖2 + ‖y‖2 + 2〈x, y〉 ≤ ‖x‖2 + ‖y‖2 + 2‖x‖‖y‖ = (‖x‖+ ‖y‖)2,

con igualdad si y solo si 〈x, y〉 ≥ 0 y existe (s, t) 6= (0, 0) tal que sx+ ty ∈ N . Estas condiciones implicanque s y t no tienen signos opuestos. Por tanto podemos conseguir que ambos sean no negativos.

La desigualdad triangular nos permite, sin usar calculo diferencial, ampliar nuestro catalogo de fun-ciones convexas.

para cada d > 0 la funcion x 7→√d2 + x2 es convexa.

Page 46: Desigualdades matemáticas clásicas · 2018-02-27 · 3 Resumen En esta memoria recogemos algunas de las desigualdades m as importantes en Matem aticas, sus or ge-nes y aplicaciones.

42 CAPITULO 3. LA DESIGUALDAD DE CAUCHY-SCHWARZ

Demostracion. Gracias a la desigualdad triangular se tiene, para todo x, y ∈ R y todo 0 ≤ t ≤ 1,√d2 + ((1− t)x+ ty)2 = ‖(d, (1− t)x+ ty)‖2

= ‖(1− t)(d, x) + t(d, y)‖2≤ ‖(1− t)(d, x)‖2 + ‖t(d, y)‖2= (1− t)‖(d, x)‖2 + t‖(d, y)‖2= (1− t)

√d2 + x2 + t

√d2 + y2.

Problema 12 (Problema de la lazada). ¿De que manera debemos atarnos un zapato para optimizar lacantidad de cuerda empleada en la lazada?

Solucion. Vamos a plantear este problema en terminos matematicos.

Tenemos 2n puntos en el plano (n ∈ N), situados n de ellos en una recta rp y los n restantes en otrarecta rq paralela a la anterior. Los denotamos (Pi)

ni=1 y (Qi)

ni=1, respectivamente.

Suponemos que los puntos de las n-tuplas estan ordenados (esto es, Pi+1 no esta en el segmento P1Pi)e imponemos que el segmento PiQi es perpendicular a las rectas rp y rq. Denotamos h la distancia entreambas rectas y di la distancia de Pi a Pi+1 (que coincide con la distancia de Qi a Qi+1). Aunque noes imprescindible en nuestro razonamiento, supondremos que las distancias entre diferentes agujeros delzapato son iguales, esto es, que existe d > 0 de modo que di = d para todo i = 1, . . .n− 1.

Una lazada es una poligonal con vertices en P1, . . . , Pn, Q1, . . . , Qn que cumple las siguientes condi-ciones:

(i) Todos los puntos P1, . . . , Pn, Q1, . . . , Qn pertenecen a la poligonal y aparecen una sola vez.

(ii) Empieza en Pn y acaba en Qn.

(iii) Si el origen de un segmento de la poligonal es un punto Pi en la recta rp, su destino es un puntoQj en la recta rq (o viceversa). Ademas en los segmentos anteriores a alcanzar uno de los puntosP1 o Q1 se tiene j ≤ i, y en los segmentos posteriores a que los dos puntos P1 y Q1 se alcancen setiene i ≤ j.

Notamos que las condiciones anteriores implican que el primer segmento sera P1Q1 o Q1P1.

Page 47: Desigualdades matemáticas clásicas · 2018-02-27 · 3 Resumen En esta memoria recogemos algunas de las desigualdades m as importantes en Matem aticas, sus or ge-nes y aplicaciones.

3.10. PROBLEMAS 43

Llamamos k1, . . . , ka a los saltos entre agujeros que la poligonal produce en la recta rp, y l1, . . . , lb alos que se producen en la recta rq. Las tuplas de enteros naturales (ki)

ai=1 y (li)

bi=1 cumplen

a∑i=1

ki =b∑i=1

li = n

y determinan unıvocamente la lazada. Observando la cantidad de segmentos perpendiculares a las rectasrp y rq y los que no lo son, nos damos cuenta de que la longitud de la cuerda la podemos expresar como

h+a∑i=1

(√h2 + k2

i d2 + (ki − 1)d

)+

b∑i=1

(√h2 + l2i d

2 + (li − 1)d

).

En el caso de que nuestra lazada siga saltos de uno en uno, esto es ki = li = 1 para i = 1,. . . , n, obtenemosque la longitud es

h+(√

h2 + d2)

2n,

que, en terminos de sucesiones (ki)ai=1 y (li)

bi=1 generales, podemos escribir como

h+a∑i=1

√h2 + d2ki +

b∑i=1

√h2 + d2li.

Por tanto, si conseguimos probar que√h2 + k2d2 + (k − 1)d ≥ k

√h2 + d2

∀k ∈ N con igualdad si y solo si k = 1, tendremos que el caso saltar de uno en uno es el optimo (el demenor longitud) y el unico optimo.

Usamos para ello que la funcion

f(x) =√d2 + x2

es estrictamente convexa.

Se tiene

k√h2 + d2 = kf(h) = kf

(1

kh+

1

k0

)≤ k

(1

kf(kh) +

k − 1

kf(0)

)= k

(1

k

√k2h2 + d2 +

k − 1

kd

)=√h2 + k2d2 + (k − 1)d,

con igualdad si y solo si k−1k = 0 (es decir, si y solo si k = 1).

3.10. Problemas

Comenzamos con una nueva prueba de una desigualdad que ya resolvimos en la seccion anteriormediante la desigualdad de medias.

Problema 13. [Olimpiada Matematica Espanola, 2009]

Sean a, b, c numeros positivos tales que abc = 1. Probad que(a

1 + ab

)2

+

(b

1 + bc

)2

+

(c

1 + ca

)2

≥ 3

4.

Page 48: Desigualdades matemáticas clásicas · 2018-02-27 · 3 Resumen En esta memoria recogemos algunas de las desigualdades m as importantes en Matem aticas, sus or ge-nes y aplicaciones.

44 CAPITULO 3. LA DESIGUALDAD DE CAUCHY-SCHWARZ

Solucion. Podemos ver la parte izquierda de la desigualdad como(a

1 + ab

)2

+

(b

1 + bc

)2

+

(c

1 + ca

)2

= ‖~x‖22 = ‖~x‖2‖~y‖2

donde

‖~x‖ =

(a

1 + ab,

b

1 + bc,

c

1 + ca

)y

‖~y‖ =

(c

1 + ca,

a

1 + ab,

b

1 + bc

).

Por la desigualdad de Cauchy-Schwarz sabemos que

‖~x‖2‖~y‖2 ≥ 〈‖~x‖, ‖~y‖〉.

Aplicando esto, tenemos que(a

1 + ab

)2

+

(b

1 + bc

)2

+

(c

1 + ca

)2

= ‖~x‖2‖~y‖2≥ 〈‖~x‖, ‖~y‖〉

=

(ac

(1 + ca)(1 + ab)

)2

+

(ab

(1 + ab)(1 + bc)

)2

+

(bc

(a+ bc)(1 + ca)

)2

=ac(1 + bc) + ab(1 + ca) + bc(1 + ab)

(1 + ab)(1 + bc)(1 + ca)

=ac+ ab+ bc+ a+ b+ c

1 + ab+ bc+ ca+ a+ b+ c+ 1.

Por tanto, basta demostrar que

ac+ ab+ bc+ a+ b+ c

1 + ab+ bc+ ca+ a+ b+ c+ 1≥ 3

4.

Operando, vemos que esta ultima desigualdad equivale a

4(ac+ ab+ bc) + 4(a+ b+ c) ≥ 6 + 3(ab+ bc+ ca) + 3(a+ b+ c),

que a se vez es equivalente aac+ ab+ bc+ a+ b+ c ≥ 6.

Teniendo en cuenta que abc = 1, esta expresion es equivalente a

(1

a+

1

b+

1

c) + a+ b+ c ≥ 6.

Lo reagrupamos y nos queda

a+1

a+ b+

1

b+ c+

1

c≥ 6.

Por el lema 6, sabemos que

a+1

a︸ ︷︷ ︸≥2

+ b+1

b︸ ︷︷ ︸≥2

+ c+1

c︸ ︷︷ ︸≥2

.

Por lo que concluimos que

a+1

a+ b+

1

b+ c+

1

c≥ 6

es cierta.

A continuacion presentamos una nueva resolucion del Problema 6, que ya hemos probado en la segundaseccion mediante la desigualdad de Jensen.

Page 49: Desigualdades matemáticas clásicas · 2018-02-27 · 3 Resumen En esta memoria recogemos algunas de las desigualdades m as importantes en Matem aticas, sus or ge-nes y aplicaciones.

3.10. PROBLEMAS 45

Problema 14. Sean a, b, c numeros reales positivos tales que abc = 1. Probad que

1

a3(b+ c)+

1

b3(c+ a)+

1

c3(a+ b)≥ 3

2.

Solucion. Hacemos el cambio x = 1a , y = 1

b , z = 1c , como hemos hecho en el problema 5. Sustituyendo y

operando obtenemosx2

y + z+

y2

x+ z+

z2

z + y≥ 3

2.

Multiplicamos por x+ y + z la parte izquierda de la desigualdad(x2

y + z+

y2

x+ z+

z2

z + y

)(x+ y + z) =

1

2((x+ y) + (y + z) + (x+ z))

(x2

y + z+

y2

x+ z+

z2

z + y

).

Aplicamos Cauchy-Schwarz

1

2((x+ y) + (y + z) + (x+ z))

(x2

y + z+

y2

x+ z+

z2

z + y

)≥ 1

2

⟨(√y + z,

√z + x,

√x+ y

),

(x√y + z

,y√z + x

,z√x+ y

)⟩=

1

2(x+ y + z)2.

Es decir, tenemos que (x2

y + z+

y2

x+ z+

z2

z + y

)≥ 1

2

(x+ y + z)2

(x+ y + z)=x+ y + z

2.

Usamos ahora la desigualdad entre las medias aritmetica y geometrica,

x+ y + z

3≥ 3√xyz.

Operamos

x+ y + z ≥ 3 3√xyz = 3

3

√1

a

1

b

1

c= 3

3

√1

abc= 3

3√

1 = 3.

Aplicamos esto a la desigualdad que tenıamos(x2

y + z+

y2

x+ z+

z2

z + y

)≥ x+ y + z

2≥ 3

2.

Por lo que (x2

y + z+

y2

x+ z+

z2

z + y

)≥ 3

2

como querıamos probar.

Problema 15. Sean a, b, c numeros positivos. Probad que

a

b(b+ c)2+

b

b(c+ a)2+

c

a(a+ b)2≥ 9

4(ab+ bc+ ca)

Solucion. Observamos que

a

b(b+ c)2=

a2

(b+c)2

ab,

b

b(c+ a)2=

b2

(c+a)2

bc,

c

a(a+ b)2=

c2

(a+b)2

ac.

Teniendo esto en cuenta podemos reescribir la parte izquierda de la desigualdad del enunciado como

a

b(b+ c)2+

b

b(c+ a)2+

c

a(a+ b)2=

a2

(b+c)2

ab+

b2

(c+a)2

bc+

c2

(a+b)2

ac.

Page 50: Desigualdades matemáticas clásicas · 2018-02-27 · 3 Resumen En esta memoria recogemos algunas de las desigualdades m as importantes en Matem aticas, sus or ge-nes y aplicaciones.

46 CAPITULO 3. LA DESIGUALDAD DE CAUCHY-SCHWARZ

Por el Lema de Titu llegamos a que

a2

(b+c)2

ab+

b2

(c+a)2

bc+

c2

(a+b)2

ac≥

( ab+c + b

c+a + ca+b )

2

ab+ bc+ ca.

Teniendo en cuenta la desigualdad de Nesbitt,

a

b+ c+

b

c+ a+

c

a+ b≥ 3

2,

concluimos que( ab+c + b

c+a + ca+b )

2

ab+ bc+ ca≥ 9

4(ab+ bc+ ca).

Pasamos a ver una nueva resolucion de la desigualdad de Nesbitt, que ya hemos probado en la segundaseccion 2 (Problema 2) y en la seccion 3 (Problema 11).

Problema 16 (Desigualdad de Nesbitt). Dados a, b, c > 0, probad que

a

b+ c+

b

a+ c+

c

a+ b≥ 3

2

Solucion. Multiplicamos numerador y denominador de cada fraccion del miembro de la izquierda pora, b, c, respectivamente

a2

ab+ ac+

b2

ab+ bc+

c2

bc+ ac.

Por el Lema de Titu llegamos a que

a2

ab+ ac+

b2

ab+ bc+

c2

bc+ ac≥ (a+ b+ c)2

2(ab+ bc+ ca).

Basta probar que2(a+ b+ c)2 ≥ 6(ab+ bc+ ca).

Simplificamos la expresion y nos queda

(a+ b+ c)2 ≥ 3(ab+ bc+ ca),

que es equivalente aa2 + b2 + c2 ≥ ab+ bc+ ca,

que es cierto para numeros reales a, b, c > 0 cualesquiera.

Page 51: Desigualdades matemáticas clásicas · 2018-02-27 · 3 Resumen En esta memoria recogemos algunas de las desigualdades m as importantes en Matem aticas, sus or ge-nes y aplicaciones.

47

Capıtulo 4

La desigualdad de Holder

Otto Ludwing Holder (Stuttgart, 22 de diciembre de 1859 – Leipzig, 29 de agosto de 1937) fue unmatematico aleman. Estudio Ingenierıa, durante un ano, en el Politecnico de Stuttgart y desde 1877estudio en la Universidad de Berlın. En 1882 presento su tesis doctoral en la Universidad de Tubingen ytrabajo en la Universidad de Leipzig desde 1899 hasta su jubilacion. Es conocido por sus contribucionestanto en algebra como en analisis matematico.

Se intereso por el algebra y, en concreto por la teorıa de grupos, influenciado por Kronecker y Klein.Probo el conocido Teorema de Jordan-Holder, investigo los grupos finitos simples y publico un artıculomostrando que los grupos finitos simples hasta orden 200 eran conocidos. En 1895 redacto un extensotrabajo sobre extensiones de grupos e introdujo los conceptos de automorfismo interno y externo.

No menos importantes fueron sus aportaciones en el analisis matematico. Investigo las funcionesanalıticas y procedimientos de suma usando medias aritmeticas. Probo su conocida desigualdad de Holdery la condicion que lleva su nombre, empleada en areas del analisis como la teorıa de las ecuacionesdiferenciales en derivadas parciales y los espacios de funciones. Recordamos que una funcion f : I → R,donde I es un intervalo, satisface una condicion de Holder de parametro α > 0 si existe una constante Ctal que

|f(y)− f(x)| ≤ C|x− y|α, x, y ∈ R.Las funciones de Holder de parametro 1 son las llamadas funciones de Lipschitz. El concepto pierde sen-tido para α > 1, pues entonces las funciones de Holder son constantes.

A partir de 1900 comenzo a interesarse por la geometrıa lineal proyectiva y mas adelante, por cuestionesfilosoficas.

4.1. Introduccion

Consideremos un espacio de medida (Ω,Σ, µ), una funcion g ∈ L1(µ) y otra funcion medible f : Ω→ Rtal que ‖f‖∞,µ := sup es |f | <∞. Entonces

|fg| ≤ ‖f‖∞,µ |f | µ-c.t.p.

Page 52: Desigualdades matemáticas clásicas · 2018-02-27 · 3 Resumen En esta memoria recogemos algunas de las desigualdades m as importantes en Matem aticas, sus or ge-nes y aplicaciones.

48 CAPITULO 4. LA DESIGUALDAD DE HOLDER

Por tanto fg ∈ L1(µ) y, ademas,∣∣∣∣∫Ω

fg dµ

∣∣∣∣ ≤ ∫Ω

|fg| dµ ≤ ‖f‖∞,µ ‖f‖1.

Este resultado tiene una relacion muy estrecha con el Teorema 11. Podrıamos decir que el Teorema 11 seapoya en que el par (2, 2) es un par de ındices conjugados mientras que el que acabamos de comentar sebasa en que el par (1,∞) es un par de ındices conjugados. En general, definimos el siguiente concepto.

Definicion 1. Sean p, q ∈ (0,∞]. Decimos que (p, q) es un par de ındices conjugados y que q es elconjugado de p si

1

p+

1

q= 1.

Comentario. Si (p, q) es un par de ındices conjugados entonces, forzosamente p, q ∈ [1,∞]. Ademaspara cada p ∈ [1,∞] existe un unico q ∈ [1,∞] tal que q es conjugado de p.

Definicion 2. Sea (Ω,Σ, µ) espacio de medida positiva y 1 ≤ p <∞. Sea f : Ω→ [0,∞] medible.

‖f‖p =

(∫Ω

fp dµ

)1/p

∈ [0,∞]

si 1 ≤ p <∞, y‖f‖p = sup es

w∈Ωf(w)

si p =∞.

En realidad, es mas correcto denotar estos calibradores mediante ‖f‖p,µ, sobre todo en el caso p =∞,en el que puede no quedar suficientemente claro si para definirlo tomamos el supremo o el supremoesencial. Sin embargo, en esta seccion unicamente manejaremos la definicion que se obtiene a partir delsupremo esencial.

4.2. Enunciado y demostracion

Comenzamos con un lema auxiliar.

Lema 7. Sean x e y numeros en [0,∞]. Sean p y q definidos como 1/p+1/q = 1, con p tal que 1 < p <∞.Entonces,

xy ≤ xp

p+yq

q, 0 ≤ x, y <∞.

Ademas, la igualdad se da si y solo si o bien x1/p = y1/q o bien uno de los numeros es ∞ y el otro es nonulo.

Demostracion. Aplicando el Lema 5 con t = 1/q, de modo que 1− t = 1/p, obtenemos

a1/pb1/q ≤ 1

pa+

1

qb, a, b ≥ 0.

Consideramos ahora a tal que x1/p = a y b tal que y1/q = b. Llegamos a

xy ≤ 1

pxp +

1

qyq,

como buscabamos. Ademas la igualdad se obtiene si y solo si x1/p = y1/q.Cuando o bien x =∞ o bien y =∞, la desigualdad es obvia.

Teorema 15 (Desigualdad de Holder para funciones positivas). Sean p y q exponentes conjugados,1 ≤ p, q ≤ ∞. Sea (Ω,Σ, µ) un espacio de medida. Sean f y g funciones medibles en Ω, con recorrido en[0,∞]. Entonces ∫

Ω

fg dµ ≤ ‖f‖p‖g‖q.

Ademas, la igualdad se produce si y solo si se dan uno de los casos siguientes.

Page 53: Desigualdades matemáticas clásicas · 2018-02-27 · 3 Resumen En esta memoria recogemos algunas de las desigualdades m as importantes en Matem aticas, sus or ge-nes y aplicaciones.

4.2. ENUNCIADO Y DEMOSTRACION 49

∫Ωfg dµ =∞.

Existe C ∈ [0,∞) tal que f1/p = Cg1/q, o existe C ∈ [0,∞) tal que g1/q = Cf1/p.

Demostracion. El razonamiento en el caso p =∞ y, por tanto q = 1, corresponde al indicado al comienzode la seccion. Ademas la igualdad se da si y solo si fg = f‖g‖∞ µ-c.t.p. Esta igualdad conduce a uno delos dos casos descritos en el enunciado

El caso p = 1 y, por tanto q =∞, es simetrico y vale el mismo razonamiento.En el caso en que 1 < p < ∞ y, por tanto 1 < q < ∞, denotamos A = ‖f‖p y B = ‖g‖q. Vamos a

distinguir tres casos.• Si A = 0, entonces f = 0 µ-c.t.p. Por tanto, fg = 0 en µ-c.t.p., y se verifica la desigualdad. Realmente,se da la igualdad

∫Ωfg dµ = ‖f‖p‖g‖q = 0.

• Si A > 0 y B = ∞, entonces ‖f‖p‖g‖q = ∞ y la desigualdad es trivial. La igualdad se produce,obviamente, cuando

∫Ωfg dµ =∞.

• Supongamos que 0 < A <∞, 0 < B <∞. Tomamos

F =f

A, G =

g

B. (4.1)

Esto proporciona ∫Ω

F p dµ =

∫Ω

Gq dµ = 1.

Se tiene que, gracias al Lema 7,

F (ω)G(ω) ≤ 1

pF (ω)p +

1

qG(ω)q

para todo ω ∈ Ω. Integrando obtenemos

1

AB

∫Ω

fg dµ =

∫Ω

FGdµ ≤ 1

p

∫Ω

F dµ+1

q

∫Ω

Gdµ =1

p+

1

q= 1,

que conduce facilmente a la desigualdad del teorema. La igualdad, en este tercer caso, se da cuandoF 1/p = G1/q µ-c.t.p. Esto conduce a f1/p = A1/pB−1/qg1/q.

La argumentacion de arriba demuestra que para que se verifique la igualdad es necesario que secumpla una de las dos condiciones del enunciado. Que la primera de ellas es suficiente es consecuencia dela desigualdad ya probada, pues, en este caso, ∞ =

∫Ωfg dµ ≤ ‖f‖p‖g‖q. Si se da el segundo caso que

propone el enunciado, o sea, f1/p = Cg1/q, se tiene∫Ω

fg dµ = Cp‖g‖q, ‖f‖p = Cp‖g‖q−1,

y, por tanto, tambien se produce la igualdad.

A continuacion, damos una version de la desigualdad de Holder para funciones con valores reales.

Definicion 3. Sea (Ω,Σ, µ) espacio de medida positiva y 1 ≤ p <∞. Sea f : Ω→ R medible.

‖f‖p =

(∫Ω

|fp| dµ)1/p

si 1 ≤ p <∞, y‖f‖p = sup es

w∈Ω|f(w)|

si p =∞.Decimos que una funcion f ∈ Lp(µ) si ‖f‖p <∞.

Definicion 4. Sea x ∈ R, denotamos por sign(x) al signo de x y lo definimos como 1, x > 00, x = 0−1, x < 0.

Page 54: Desigualdades matemáticas clásicas · 2018-02-27 · 3 Resumen En esta memoria recogemos algunas de las desigualdades m as importantes en Matem aticas, sus or ge-nes y aplicaciones.

50 CAPITULO 4. LA DESIGUALDAD DE HOLDER

Teorema 16 (Desigualdad de Holder). Si p y q son exponentes conjugados, 1 ≤ p ≤ ∞, y si f ∈ Lp(µ)y g ∈ Lq(µ), entonces fg ∈ L1(µ), y

‖fg‖1 ≤ ‖f‖p‖g‖q. (4.2)

Ademas, la igualdad se da si y solo si sign(f) = sign(g) µ-c.t.p. y, ademas, o bien existe C ∈ [0,∞) talque |f |1/p = C|g|1/q, o bien existe C ∈ [0,∞) tal que |g|1/q = C|f |1/p.

Demostracion. Aplicando el Teorema 15 a |f | y |g|. Obtenemos que∫Ω

|fg| dµ =

∫Ω

|f ||g| dµ ≤ ‖|f |‖p‖|g|‖q = ‖f‖p‖g‖q <∞.

Puesto que fg es medible, esto nos da que fg ∈ L1(µ). Ademas,∣∣∣∣∫Ω

fg dµ

∣∣∣∣ ≤ ∫Ω

|fg| dµ ≤ ‖f‖p‖g‖q <∞.

La igualdad se produce si y solo si suceden, simultaneamente, fg = |fg| y |f |1/p = C|g|1/q para algunaconstante C ∈ [0,∞) (o a la inversa). Notamos que la primera condicion equivale a sign(f) = sign(g)µ-c.t.p

Comentario. Si aplicamos el Teorema 15 en el caso en que p > 1, µ(Ω) = 1 y f ≡ 1, obtenemos ladesigualdad ∫

Ω

g dµ ≤(∫

Ω

1 dµ

)1/p(∫Ω

gq dµ

)1/q

=

(∫Ω

gq dµ

)1/q

.

con p ≥ 1.

Si ahora tomamos f medible positiva 1 ≤ a < b < ∞, podemos elegir p tal que b = aq. Si aplicamos loanterior a g = fa obtenemos ∫

Ω

fa dµ ≤(∫

Ω

faq dµ

)1/q

=

(∫Ω

f b dµ

)a/b.

Elevando a 1/a llegamos a (∫Ω

fa dµ

)1/a

︸ ︷︷ ︸Ma

≤(∫

Ω

f b dµ

)1/b

︸ ︷︷ ︸Mb

.

O sea, llegamos a la desigualdad entre la media de orden a y la media de orden b. El decir, la desigualdadde medias (Teorema 5) es consecuencia de la desigualdad de Holder.

Comentario. En el caso en que p = 2 y, por tanto q = 2, la desigualdad de Holder, en la version queda el Teorema 16, se convierte exactamente en el Teorema 11.

Por lo tanto, podemos decir que en un cierto sentido, la desigualdad de Holder implica la desigualdadde Cauchy-Schwarz.

Page 55: Desigualdades matemáticas clásicas · 2018-02-27 · 3 Resumen En esta memoria recogemos algunas de las desigualdades m as importantes en Matem aticas, sus or ge-nes y aplicaciones.

51

Capıtulo 5

La desigualdad de reordenamiento

No resulta sencillo encontrar el origen de esta desigualdad. Aunque no cabe duda de que por susencillez era conocida mucho antes, su enunciado aparece por primera vez, al menos hasta donde alcanzanuestro conocimiento, en el manual sobre desigualdades [6].

5.1. Introduccion y enunciado

Sean (ai)ni=1, (bi)

ni=1 n-tuplas de numeros positivos. Realizamos sumas de productos de elementos de

una n-tupla contra elementos de la otra. O sea,

aα(1)bβ(1) + · · ·+ aα(i)bβ(i) + · · ·+ aα(n)bβ(n) =

n∑i=1

aα(i)bβ(i) (5.1)

donde α, β son permutaciones del conjunto 1, . . . , i, . . . , n; o sea, biyecciones de este conjunto en sı mis-mo.

Nos preguntamos por el mayor y menor valor que alcanza (5.1). La respuesta, como veremos, es queel mayor se alcanza cuando ambas n-tuplas siguen el mismo orden con el criterio de mayor a menor. Elmınimo, cuando siguen ordenes opuestos.

Demos un enunciado preciso:

Teorema 17. Sean (ai)ni=1, (bi)

ni=1 tales que

a1 > a2 > . . . > an ≥ 0b1 > b2 > . . . > bn ≥ 0.

Entoncesn∑i=1

aibn+1−i ≤n∑i=1

aibβ(i) ≤n∑i=1

aibi, (5.2)

donde β es una permutacion de 1, . . . , i, . . . , n. La segunda desigualdad se convierte en identidad cuandoy solo cuando β es la identidad. La primera desigualdad se convierte en identidad cuando y solo cuandoβ viene dada por β(i) = n+ i− 1.

5.2. Demostracion

Vamos a demostrar la segunda desigualdad por el metodo de induccion. Remarcamos que la demos-tracion de la primera es totalmente analoga.

Demostracion. • Base de induccion. Para n = 2, por hipotesis del Teorema 17 tenemos que a1 > a2 yb1 > b2, es decir,

(a2 − a1)(b2 − b1) ≥ 0.

Page 56: Desigualdades matemáticas clásicas · 2018-02-27 · 3 Resumen En esta memoria recogemos algunas de las desigualdades m as importantes en Matem aticas, sus or ge-nes y aplicaciones.

52 CAPITULO 5. LA DESIGUALDAD DE REORDENAMIENTO

Operando llegamos a quea2b2 − a1b2 − a2b1 + a1b1 ≥ 0,

o lo que es lo mismo,a1b1 + a2b2 ≥ a1b2 + a2b1,

como querıamos probar (con igualdad cuando a1 = a2 y b1 = b2).

• Paso de induccion. Suponemos que es cierto para un cierto numero natural n. Veamos que ocurrepara n+ 1.En este caso la desigualdad (5.2) es de la forma

a1bβ(1) + · · ·+ anbβ(n) + an+1bβ(n+1) ≤ a1b1 + · · ·+ anbn + an+1bn+1.

Sea i tal que β(i) = n+ 1. Tenemos que β(i) > β(n+ 1) y por lo tanto, bβ(i) ≥ bβ(n+1). Ademas sabemosque ai ≤ an+1. Por lo que

0 ≤ (an+1 − ai)(bβ(i) − bβ(n+1)).

Observamos queaibn+1 + an+1bβ(n+1) ≤ aibβ(n+1) + an+1bn+1.

De lo anterior deducimos que

a1bβ(1) + · · ·+ aibn+1 + · · ·+ an+1bβ(n+1) < a1bβ(1) + · · ·+ aibβ(n+1) + · · ·+ an+1bn+1.

Aplicando la hipotesis de induccion,

a1bβ(1) + · · ·+ aibβ(n) + · · ·+ anbβ(n) ≤ a1b1 + a2b2 + · · ·+ anbn.

De esta manera llegamos a

a1bβ(1) + a2bβ(2) + · · ·+ anbβ(n) + an+1bβ(n+1) ≤ a1b1 + a2b2 + · · ·+ anbn + an+1bn+1,

como querıamos probar.

Comentario. En el anterior teorema, si imponemos que los terminos de la n-tupla considerados sonsolamente decrecientes en lugar de estrictamente decrecientes, es mas tedioso describir cuando se producela igualdad en la desigualdad. Pero, por supuesto, la desigualdad sigue siendo valida.

Corolario 2. Sean (ai)ni=1, (bi)

ni=1 tales que

a1 ≥ a2 ≥ · · · ≥ an ≥ 0b1 ≥ b2 ≥ · · · ≥ bn ≥ 0.

Entoncesn∑i=1

aibn+1−i ≤n∑i=1

aibβ(i) ≤n∑i=1

aibi,

donde α, β son permutaciones de 1, . . . , i, . . . , n.

Demostracion. Sea β una permutacion y sea ε > 0, realizamos una adecuada pequena alteracion de lasn-tuplas del enunciado. Es decir, notamos que existen

a1 > a2 > . . . > an ≥ 0

b1 > b2 > . . . > bn ≥ 0

tales que ∣∣∣∣∣n∑i=1

aibβ(i) −n∑i=1

aibβ(i)

∣∣∣∣∣ < ε.

Por el Teorema 17,n∑i=1

aibβ(i) ≤n∑i=1

aibi.

Page 57: Desigualdades matemáticas clásicas · 2018-02-27 · 3 Resumen En esta memoria recogemos algunas de las desigualdades m as importantes en Matem aticas, sus or ge-nes y aplicaciones.

5.3. PROBLEMAS 53

Por lo tanto,n∑i=1

aibβ(i) ≤n∑i=1

aibi + 2ε.

Como ε puede tomarse arbitrariamente pequeno,

n∑i=1

aibβ(i) ≤n∑i=1

aibi.

De manera analoga se demuestra la primera desigualdad.

Comentario. Existe una version de la desigualdad de reordemiento en el ambiente de la teorıa abstractade integracion. Dado el espacio de medida (Ω,Σ, µ), denotamos por L0(µ) el espacio de funciones mediblesf tales que

Df (t) := µ(ω ∈ Ω: |f(ω)| > t) <∞, 0 < t <∞.

Df se llama funcion de distribucion de f . Puede demostrarse que dada una funcion positiva f ∈ L0(µ),existe una unica funcion f∗ : [0, µ(Ω)) → [0,∞) decreciente continua a derecha con la misma funcion dedistribucion que f , es decir,

µ(ω ∈ Ω: |f(ω)| > t) = |(0 ≤ x < µ(Ω): |f∗(x)| > t)|, 0 < t <∞,

donde | · | denota la medida de Lebesgue. Tal funcion f∗ se llama reordenada decreciente de f .Con esta notacion se tiene, para f , g ∈ L0(µ) positivas,∫

Ω

fg dµ ≤∫ µ(Ω)

0

f∗(x)g∗(x) dx.

(Ver [2] para mas detalle). Supongamos que tenemos dos sucesiones

a1 ≥ a2 ≥ · · · ≥ an ≥ 0,b1 ≥ b2 ≥ · · · ≥ bn ≥ 0,

y una permutacion β. Pensamos en las n-tuplas f = (ai)ni=1, g = (bβ(i))

ni=1 como funciones en el espacio

de medida que proporciona la medida contadora µ en Ω = 1, . . . , n. Entonces,

f∗ =n∑i=1

aiχ[i−1,i), g∗ =n∑i=1

biχ[i−1,i).

Por tanto, ∫Ω

fg dµ =n∑i=1

aibβ(i),

∫ µ(Ω)

0

f∗(x)g∗(x) dx =n∑i=1

aibi.

Recuperamos ası una de las desigualdades que nos proporciona el Corolario 2.

5.3. Problemas

Problema 17. Sean a, b, c numeros positivos. Probad que

a+ b+ c ≤ a2 + b2

2c+b2 + c2

2a+c2 + a2

2b≤ a3

bc+b3

ca+c3

ab.

Solucion. Podemos suponer sin perdida de generalidad, a ≤ b ≤ c. Esto implica que a2 ≤ b2 ≤ c2 y1/a ≥ 1/b ≥ 1/c. De este modo tenemos que

a+ b+ c = a2 · 1

a+ b2 · 1

b+ c2 · 1

c≤ b2 · 1

a+ c2 · 1

b+ a2 · 1

c,

y tambien,

a+ b+ c ≤ c2 · 1

a+ a2 · 1

b+ b2 · 1

c.

Page 58: Desigualdades matemáticas clásicas · 2018-02-27 · 3 Resumen En esta memoria recogemos algunas de las desigualdades m as importantes en Matem aticas, sus or ge-nes y aplicaciones.

54 CAPITULO 5. LA DESIGUALDAD DE REORDENAMIENTO

Sumando las dos desigualdades anteriores y dividiendo entre 2 llegamos a

a+ b+ c ≤ a2 + b2

2c+b2 + c2

2a+c2 + a2

2b,

quedando probada la primera desigualdad del enunciado.Para la segunda, observamos que a ≤ b ≤ c tambien implica que a3 ≤ b3 ≤ c3 y que 1/bc ≤ 1/ca ≤ 1/ab.Tenemos pues,

a3

bc+b3

ca+c3

ab= a3 · 1

bc+ b3 · 1

ca+ c3 · 1

ab≥ b3 · 1

bc+ c3 · 1

ca+ a3 · 1

ab=b2

c+c2

a+a2

b,

y tambien,a3

bc+b3

ca+c3

ab≥ c3 · 1

bc+ a3 · 1

ca+ b3 · 1

ab=c2

b+a2

c+b2

a.

Sumando estas dos ultimas desigualdades,

2

(a3

bc+b3

ca+c3

ab

)≥ a2 + b2

c+b2 + c2

a+c2 + a2

b,

que prueba la segunda desigualdad.

Page 59: Desigualdades matemáticas clásicas · 2018-02-27 · 3 Resumen En esta memoria recogemos algunas de las desigualdades m as importantes en Matem aticas, sus or ge-nes y aplicaciones.

55

Capıtulo 6

La desigualdad de Chebyshev

Pafnuty Lvovich Chebyschov (Okatovo, 16 de mayo de 1821 – San Petesburgo, 8 de diciembre de1894) es conocido principalmente por su trabajo en las areas de probabilidad y estadıstica.

Comenzo sus estudios universitarios de Matematicas en 1837 y cuatro anos despues, los termino des-tacando por su nivel. En 1846 defendio su tesis Un intento de analisis elemental de la teorıa probabilıstica.Fue profesor de matematicas en San Petesburgo y fundo una importante escuela de matematicos en estaciudad. Mas adelante se convirtio en miembro de la Academia Imperial de Ciencias y posteriormente, fueelegido miembro honorario de la Sociedad Matematica de San Petesburgo (1893).

Segun menciono el mismo, fue su profesora de musica de la infancia la que “llevo su mente a laexactitud y el analisis.”

Se dedico al estudio de la teorıa de los numeros y el calculo de probabilidades. Entre sus aportacionesmas notables destacan la generalizacion de la Ley de los grandes numeros, el Teorema del lımite centraly las desigualdades que llevan su nombre. Ademas de la que incluimos en este manuscrito, recordamos,siendo (Ω,Σ, µ) un espacio de medida, la que en teorıa de integracion abstracta se enuncia como ladesigualdad de Chebyshev,

µ(ω ∈ Ω: f(ω) ≥ t) ≤ 1

t

∫Ω

f dµ, f ≥ 0 medible. (6.1)

La desigualdad (6), aunque sencilla de demostrar (basta comparar f con la funcion tχA, con A = ω ∈Ω: f(ω) ≥ t), tiene importantes aplicaciones en Analisis Matematico. En Teorıa de la Probabilidad seconoce como desigualdad de Chebyshev la que afirma que, si µ es una medida de probabilidad,

µ(ω ∈ Ω: |f(ω)−m| ≥ tσ) ≤ 1

t2, f variable aleatoria de media m y desviacion tıpica σ,

que es una sencilla consecuencia de (6).

Page 60: Desigualdades matemáticas clásicas · 2018-02-27 · 3 Resumen En esta memoria recogemos algunas de las desigualdades m as importantes en Matem aticas, sus or ge-nes y aplicaciones.

56 CAPITULO 6. LA DESIGUALDAD DE CHEBYSHEV

Remarcamos que Chebyshev es una de las posibles maneras de trasladar del alfabeto cirılico el apellidode este matematico ruso.

6.1. Desigualdad de Chebyshev sobre sumas

Teorema 18. Sean a1 ≥ a2 ≥ · · · ≥ an y b1 ≥ b2 ≥ · · · ≥ bn, entonces

n

(n∑i=1

aibi

)≥

(n∑i=1

ai

)(n∑i=1

bi

).

Si, en cambio, tenemos que a1 ≥ a2 ≥ · · · ≥ an y bn ≥ bn−1 ≥ · · · ≥ b1, entonces

n

(n∑i=1

aibi

)≤

(n∑i=1

ai

)(n∑i=1

bi

).

La igualdad en ambos casos se alcanza para el caso n = 1.

Esta desigualdad de Chebyschev es una consecuencia de la desigualdad de reordenamiento. Podemosverlo en la siguiente demostracion:

Demostracion. Por la desigualdad de reordenamiento, sabemos que la suma

S = a1bi1 + a2bi2 + · · ·+ anbin

es maxima cuando ik = k, es decir, cuando se tiene que S = a1b1 + · · ·+ anbn.Teniendo esto en cuenta, es obvio que

n∑i=1

aibi = a1b1 + · · ·+ anbn = a1b1 + a2b2 + · · ·+ anbn.

n∑i=1

aibi = a1b1 + · · ·+ anbn ≥ a1b2 + a2b3 + · · ·+ anb1.

n∑i=1

aibi = a1b1 + · · ·+ anbn ≥ a1b3 + a2b4 + · · ·+ anb2.

. . .n∑i=1

aibi = a1b1 + · · ·+ anbn ≥ a1bn + a2b1 + · · ·+ anbn−1.

Sumando todas estas desigualdades, obtenemos

n(a1b1 + · · ·+ anbn) ≥ (a1 + · · ·+ an)(b1 + · · ·+ bn)

que equivale a

n

(n∑i=1

aibi

)≥

(n∑i=1

ai

)(n∑i=1

bi

)como querıamos probar.

De manera analoga, se prueba la segunda desigualdad enunciada teniendo en cuenta que la suma

S = a1bi1 + a2bi2 + · · ·+ anbin

es maxima cuando ik = n+ 1− k, es decir, cuando se tiene que S = a1bn + · · ·+ anb1.

Page 61: Desigualdades matemáticas clásicas · 2018-02-27 · 3 Resumen En esta memoria recogemos algunas de las desigualdades m as importantes en Matem aticas, sus or ge-nes y aplicaciones.

6.2. PROBLEMAS 57

6.2. Problemas

Problema 18. Probar que si x, y, z son numeros positivos tales que xyz = 1 entonces

x5y5

x2 + y2+

y5z5

y2 + z2+

x5z5

x2 + z2≥ 3

2.

Solucion. Hacemos el cambio x = 1/a, y = 1/b, z = 1/c de manera que abc = 1. Sustituimos y operamos

x5y5

x2 + y2+

y5z5

y2 + z2+

x5z5

x2 + z2=

1

a3b3(b2 + a2)+

1

b3c3(c2 + b2)+

1

c3a3(a2 + c2)

=c3

a2 + b2+

a3

b2 + c2+

b3

a2 + c2.

Sin perdida de generalidad podemos suponer a ≤ b ≤ c de modo que

a2

b2 + c2≤ b2

c2 + a2≤ c2

a2 + b2.

Por la desigualdad de Chebyshev para sumas llegamos a

a3

b2 + c2+

b3

c2 + a2+

c3

a2 + b2≥ 1

3(a+ b+ c)

(a2

b2 + c2+

b2

c2 + a2+

c2

a2 + b2

).

Notemos que la parte derecha de la desigualdad es mayor o igual que 3/2, por tratarse de la desigualdadde Nesbitt, como querıamos probar.

Page 62: Desigualdades matemáticas clásicas · 2018-02-27 · 3 Resumen En esta memoria recogemos algunas de las desigualdades m as importantes en Matem aticas, sus or ge-nes y aplicaciones.
Page 63: Desigualdades matemáticas clásicas · 2018-02-27 · 3 Resumen En esta memoria recogemos algunas de las desigualdades m as importantes en Matem aticas, sus or ge-nes y aplicaciones.

59

Capıtulo 7

Desigualdad de Muirhead

Robert Franklin Muirhead (Glasgow, 1860 – 1941) fue un matematico escoces. Estudio Matematicasen la Universidad de Glasgow, donde se graduo con excelentes resultados. Continuo sus estudios en laUniversidad Santa Catarina de Cambridge. Allı gano en 1886 el Premio Smith por su ensayo sobre Lasleyes del movimiento de Newton.

Fue elegido miembro de la Sociedad Matematica de Edimburgo en 1884 y fue nombrado MiembroHonorıfico de esta Sociedad en 1912. Publico varios artıculos en diferentes revistas pero su fama se debea la desigualdad que trataremos a continuacion: la desigualdad de Muirhead.

7.1. Introduccion

Sean α = (α1, . . . , αn) y β = (β1, . . . , βn) dos n-tuplas decrecientes positivas, o sea, tales que

α1 ≥ α2 ≥ · · · ≥ αn ≥ 0,β1 ≥ β2 ≥ · · · ≥ βn ≥ 0.

Diremos que α β (α mayoriza a β) si α1 + · · ·+ αn = β1 + · · ·+ βn, y ademas se cumple que

k∑i=1

αi ≥k∑i=1

βi.

Dado el vector x = (x1, . . . , xn) donde x1, . . . , xn ≥ 0, definimos la suma simetrica

Fα(x1, . . . , xn) =∑π∈Sn

xα1

π(1)xα2

π(2) · · ·xαnπ(n),

donde Sn es el grupo simetrico sobre n elementos, o sea, el conjunto de permutaciones de (1, . . . , i, . . . , n).Para esta definicion utilizamos el convenio x0 = 1 para todo x ≥ 0 (en particular, 00 = 1).

Page 64: Desigualdades matemáticas clásicas · 2018-02-27 · 3 Resumen En esta memoria recogemos algunas de las desigualdades m as importantes en Matem aticas, sus or ge-nes y aplicaciones.

60 CAPITULO 7. DESIGUALDAD DE MUIRHEAD

7.2. El Teorema de Muirhead

Teorema 19 (Teorema de Muirhead). Sean α y β dos n-tuplas decrecientes positivas tales que α β,se tiene que

(a) Siempre que x1, . . . , xn ≥ 0,

Fα (x1, . . . , xn) ≥ Fβ (x1, . . . , xn) . (7.1)

(b) Si x1, . . . , xn > 0 no todos iguales y α 6= β, (7.1) produce una desigualdad estricta.

Antes de abordar la demostracion veamos un lema sobre funciones convexas.

Lema 8. Sea ϕ : I → R una funcion convexa. Sean x, y ∈ I, se tiene que

ϕx,y : [0, 1]→ R, λ 7→ ϕx,y(λ) = ϕ((1− λ)x+ λy) + ϕ((1− λ)y + λx)

es una funcion decreciente en [0, 1/2] y creciente en [1/2, 1]. Ademas, si x 6= y y ϕ es estrictamenteconvexa, entonces, el crecimiento y decrecimiento de ϕx,y en ambas zonas es estricto.

Demostracion. Sean x, y ∈ I y λ ∈ [0, 1]. Debemos demostrar que si µ es un punto interior al segmentoque une λ con 1 − λ entonces ϕx,y(µ) ≤ ϕx,y(λ), con desigualdad estricta en los casos descritos en elenunciado.

Tomamos 0 < a < 1 tal que aλ+ (1− a)(1− λ) = µ. Notamos que entonces,

(1− a)λ+ a(1− λ) = 1− µ.

Tenemosϕx,y(λ) =(1− a)ϕ((1− λ)x+ λy) + aϕ((1− λ)y + λx)

+ aϕ((1− λ)x+ λy) + (1− a)ϕ((1− λ)y + λx)

≥ϕ((1− a)((1− λ)x+ λy) + a((1− λ)y + λx))

+ ϕ((1− a)((1− λ)y + λx) + a((1− λ)x+ λy))

=ϕx,y(µ).

En caso de que x 6= y y ϕ sea estrictamente convexa, las dos desigualdades de convexidad utilizadasarriba son estrictas.

Demostracion del Teorema 19. Utilizaremos el metodo de induccion.• Base de induccion. Para n = 2, tenemos que la desigualdad (7.1) para x1, x2 ≥ 0, es de la forma

xα11 xα2

2 + xα12 xα2

1 ≥ xβ1

1 xβ2

2 + xβ1

2 xβ2

1 ,

donde, α1, α2, β1, β2 cumplen α1 + α2 = β1 + β2, α1 ≥ β1. Todo esto equivale a la existencia de t ≥ 0 y1 ≥ λ ≥ µ ≥ 1/2 tales que

α1 = λt, α2 = (1− λ)t, β1 = µt, β2 = (1− µ)t.

Si x1, x2 > 0, existen x, y ∈ R tales que

xt1 = ex, xt2 = ey.

Usando la propiedad multiplicativa de la funcion exponencial, el lema 8 nos da, puesto que la funcionexponencial es estrictamente simetrica,

xα11 xα2

2 + xα12 xα2

1 = e(1−λ)xeλy + e(1−λ)yeλx

= e(1−λ)x+λy + e(1−λ)y+λx

≥ e(1−µ)x+µy + e(1−µ)y+µx

= e(1−µ)xeµy + e(1−µ)yeµx

≥ xβ1

1 xβ2

2 + xβ1

2 xβ2

1 .

Page 65: Desigualdades matemáticas clásicas · 2018-02-27 · 3 Resumen En esta memoria recogemos algunas de las desigualdades m as importantes en Matem aticas, sus or ge-nes y aplicaciones.

7.2. EL TEOREMA DE MUIRHEAD 61

En caso de que (α1, α2) 6= (β1, β2) tenemos λ > µ. En caso de que x1 6= x2 tenemos x 6= y. Por tanto, ladesigualdad es estricta.

En caso de que o bien x1 = 0 o x2 = 0, la desigualdad es clara.

• Paso de induccion. Suponemos que se cumple para cualquier numero natural n ≥ 2. Veamos si secumple para n+ 1. Consideramos los vectores:

(α1, . . . , αn, αn+1) (β1, . . . , βn, βn+1).

Podemos dividir el problema en dos casos:

Caso previo. Suponemos que las sucesiones tienen un elemento en comun.Sea αj = βj := γ tal elemento, tenemos que

Fα (x1, . . . , xn+1) =∑

π∈Sn+1

n+1∏i=1

xαiπ(i)

=n+1∑k=1

∑π∈Sn+1

π(j)=k

n+1∏i=1

xαiπ(i)

=n+1∑k=1

xγk

∑π∈Sn+1

π(j)=k

n+1∏i=1i6=j

xαiπ(i)

=

n+1∑k=1

xγkFαj (xk)

donde αj = (α1, . . . , αj−1, αj+1, . . . , αn) y xk = (x1, . . . , xk−1, xk+1, . . . , xn).

Este mismo logro, aplicado a la sucesion β, nos da

Fβ (x1, . . . , xn+1) =n+1∑k=1

xγkFβj (xk) .

Notamos que αj βj . Luego

Fαj (xk) ≥ Fβj (xk) ∀k = 1, . . . , n.

Sumando llegamos a queFα (x1, . . . , xn+1) ≥ Fβ (x1, . . . , xn+1) .

En caso de que α 6= β, entonces αj 6= βj . Si, ademas, la (n + 1)-tupla no es constante, existe k talque la n-tupla xk no es constante. Por tanto, si xi > 0 para todo i = 1,. . .n, Fαj (xk) > Fβj (xk). En

consecuencia, al sumar, obtenemos

Fα (x1, . . . , xn+1) > Fβ (x1, . . . , xn+1) .

Caso general. Siempre existe 1 ≤ j ≤ n tal que

αj ≥ βj , αj+1 ≥ βj+1.

Si no fuera ası, como α1 ≥ β1, tendrıamos α2 > β2, α3 > β3, . . . , αn+1 > βn+1. Luego

α1 + · · ·+ αn+1 > β1 + · · ·+ βn+1,

y llegamos a un absurdo.Ası pues, tenemos que para el elemento j ∈ 1, . . . , n, n+ 1 elegido, αj ≥ βj ≥ βj+1 ≥ αj+1.

Vamos a proponer dos modificaciones de la (n+ 1)-tupla (α1, . . . , αj , αj+1, . . . , αn+1):

Page 66: Desigualdades matemáticas clásicas · 2018-02-27 · 3 Resumen En esta memoria recogemos algunas de las desigualdades m as importantes en Matem aticas, sus or ge-nes y aplicaciones.

62 CAPITULO 7. DESIGUALDAD DE MUIRHEAD

(i) Cambiamos, en los lugares (j, j+ 1), el par (αj , αj+1) por el par (βj , αj +αj+1−βj), y dejamos comoestaban los demas terminos.

Ası, todos los terminos de la (n + 1)-tupla siguen siendo no negativos, y la suma de todos ellos nocambia. Sin embargo, no esta claro que la nueva (n+ 1)-tupla sea decreciente. Para que lo sea tiene quesuceder que

βj ≥ αj + αj+1 − βj .

O sea,

βj ≥αj + αj+1

2. (7.2)

(ii) Cambiamos, en los lugares (j, j+ 1), el par (αj , αj+1) por el par (αj +αj+1−βj+1, βj+1), y dejamoscomo estaban los demas terminos.De nuevo todos los terminos de la (n+ 1)-tupla siguen siendo no negativos, y la suma de todos ellos nocambia. Para que sea decreciente tiene que ocurrir que

αj + αj+1 − βj+1 ≥ βj+1.

O sea,αj + αj+1

2≥ βj+1. (7.3)

Notamos que o bien sucede (7.2) o bien sucede (7.3). De no ser ası, tendrıamos que

βj <αj + αj+1

2< βj+1,

y llegarıamos a la contradiccion βj < βj+1.Si se verifica (7.2), tomamos la (n+ 1)-tupla γ que se obtiene mediante el paso (i), mientras que si se

verifica (7.3), tomamos la que se obtiene mediante el paso (ii). En cualquier caso se tiene que

α tiene elementos en comun con γ (solo modificados dos de ellos y el numero total de elementos esn+ 1 ≥ 3).

γ tiene elementos en comun con β (o bien el elemento j-esimo o bien el j + 1-esimo).

Ademas, α γ β. Por lo que, aplicando el Caso previo, podemos concluir que

Fα (x1, . . . , xn+1) ≥ Fγ (x1, . . . , xn+1) ≥ Fβ (x1, . . . , xn+1) .

Si xi > 0 para todo i, no todos iguales, y ademas, α 6= β, entonces o bien α 6= γ o bien γ 6= β. Portanto, una de las dos desigualdades descritas arriba es estricta y, en consecuencia,

Fα (x1, . . . , xn+1) > Fβ (x1, . . . , xn+1) .

7.3. Problemas

Muchas desigualdades son consecuencia de esta desigualdad. Como ejemplo, vamos a probar la de-sigualdad entre las medias aritmetica y geometrica usando la desigualdad de Muirhead.

Problema 19. Probad la desigualdad entre la media aritmetca y geometrica, es decir, que

a1 + a2 + · · ·+ ann

≥ n√a1a2 . . . an

con a1, a2, . . . , an ≥ 0.

Page 67: Desigualdades matemáticas clásicas · 2018-02-27 · 3 Resumen En esta memoria recogemos algunas de las desigualdades m as importantes en Matem aticas, sus or ge-nes y aplicaciones.

7.3. PROBLEMAS 63

Solucion. Realizando el cambio de variable xi = n√ai, vemos que esta desigualdad es equivalente a

n∑i=1

ani ≥ n · a1a2 . . . an, xi ≥ 0.

Observamos quen∑i=1

ani = n · Fα(a1, . . . , an), con α = (n, 0, 0, . . . , 0)

yn · a1a2 . . . an = Fβ(a1, . . . , an), con β = (1, 1, . . . , 1).

Notamos que[n, 0, 0, . . . , 0] [1, 1, . . . , 1],

de manera que la desigualdad AM ≥ GM queda probada como consecuencia de la desigualdad deMuirhead.

Problema 20. Dados a, b, c > 0, probad que

(a+ b)(b+ c)(c+ a) ≥ 8abc.

Solucion. Desarrollando y operando ambos lados de la desigualdad llegamos a

a2b+ a2c+ b2c+ b2a+ c2a+ c2b ≥ 6abc.

Esto es equivalente a

Fα(a, b, c) ≥ Fβ(a, b, c), con α = (2, 1, 0) y β = (1, 1, 1),

que es cierto por el Teorema de Muirhead, ya que (2, 1, 0) (1, 1, 1).

Problema 21. Sean a, b, c numeros positivos tal que abc=1, probad que

1

a3(b+ c)+

1

b3(c+ a)+

1

c3(a+ b)≥ 3

2.

Nota. Antes de pasar a resolver este problema observamos que cuando el producto de x1, x2, . . . , xn es1, entonces

Fα(x1, x2, · · · , xn) = Fβ(x1, x2, · · · , xn),

con α = (α1, α2, · · · , αn) y β = (α1−r, α2−r, · · · , αn−r), donde el vector α es el conjunto de exponentes,y r ∈ R.

Solucion. Operando vemos que la desigualdad se expresa tambien como

2[b3c3(c+ a)(a+ b) + a3c3(b+ c)(c+ a) + a3b3(b+ c)(c+ a)]

2[a3b3c3(b+ c)(c+ a)(a+ b)]≥ 3[a3b3c3(b+ c)(c+ a)(a+ b)]

2[a3b3c3(b+ c)(c+ a)(a+ b)].

Quitando denominadores obtenemos la desigualdad equivalente

2[b3c3(c+ a)(a+ b) + a3c3(b+ c)(c+ a) + a3b3(b+ c)(c+ a)] ≥ 3[a3b3c3(b+ c)(c+ a)(a+ b)],

y desarrollando la expresion del numerador,

2(ab3c4+a2b3c3 + b4c4 + ab4c3 + a4bc3 + a4c4 + a3b2c3 + a3bc4 + a3b4c+ a3b3c2 + a4b4 + a4b3c) ≥≥ 3(a4b4c3 + c4b5c3 + a5b3c4 + a4b4c4 + a4b4c4 + a3b5c4 + a4b3c5 + a3b4c5).

Agrupamos terminos

2(a4b4 + b4c4 + c4a4) + 2(a4b3c+ a4c3b+ b4c3a+ b4a3c+ c4a3b+ c4b3a) + 2(a3b3c2 + b3c3a2 + c3a3b2) ≥≥ 3(a5b4c3 + a5c4b3 + b5c4a3 + b5a4c3 + c5a4b3 + c5b4c3) + 6a4b4c4.

Page 68: Desigualdades matemáticas clásicas · 2018-02-27 · 3 Resumen En esta memoria recogemos algunas de las desigualdades m as importantes en Matem aticas, sus or ge-nes y aplicaciones.

64 CAPITULO 7. DESIGUALDAD DE MUIRHEAD

Esto es equivalente a

Fα(a, b, c) + 2 · Fβ(a, b, c) + Fγ(a, b, c) ≥ 3 · Fδ(a, b, c) + Fε(a, b, c),

con α = (4, 4, 0), β = (4, 3, 1), γ = (3, 3, 2), δ = (5, 4, 3) y ε = (4, 4, 4). Observamos que

4 + 4 + 0 = 4 + 3 + 1 = 3 + 3 + 2 = 8

pero5 + 4 + 3 = 4 + 4 + 4 = 12.

Para poder aplicar la desigualdad de Muirhead, los exponentes deben sumar lo mismo, ası que tomandor = 4/3 y siguiendo la nota anterior, llegamos a

Fδ(a, b, c) = Fζ(a, b, c),

con ζ = (11/3, 8/3, 5/3) yFε(a, b, c) = Fη(a, b, c),

con η = (8/3, 8/3, 8/3).Puesto que

(4, 4, 0) (11/3, 8/3, 5/3),

(4, 3, 1) (11/3, 8/3, 5/3),

(3, 3, 2) (8/3, 8/3, 8/3),

entonces, gracias a la desigualdad de Muirhead,

Fα(a, b, c) ≥ Fζ(a, b, c),Fβ(a, b, c) ≥ Fζ(a, b, c),Fγ(a, b, c) ≥ Fη(a, b, c).

Sumando estas tres desigualdades obtenemos la desigualdad requerida.

Page 69: Desigualdades matemáticas clásicas · 2018-02-27 · 3 Resumen En esta memoria recogemos algunas de las desigualdades m as importantes en Matem aticas, sus or ge-nes y aplicaciones.

65

Capıtulo 8

La desigualdad de Schur

Issai Schur (Bielorrusia, 10 de enero de 1875 - Israel, 10 de enero de 1941), estudiante de FerdinandGeorg Frobenius, fue conocido principalmente por su trabajo en representaciones de grupos, aunquetambien trabajo en combinatoria y fısica teorica.

8.1. Introduccion

Partimos de la suma cıclica de una funcion f de n variables, definida como∑cicl

f(x1, x2, · · · , xn) =n∑k=1

f(xck(1), xck

(2), · · · , xck

(n)),

siendo c ∈ Sn una permutacion con cn la identidad. La desigualdad de Schur determina el signo de unasuma cıclica asociada a la funcion de 3 variables definida mediante

f(x, y, z) = xp(x− y)(y − z),

donde p ≥ 1 y las variables x, y, z son no negativas.

8.2. Enunciado y demostracion

Teorema 20. (Desigualdad de Schur) Sean a, b, c ≥ 0 y p ≥ 1, probad que

ap(a− b)(a− c) + bp(b− a)(b− c) + cp(c− a)(c− b) ≥ 0.

ademas, la desigualdad es estricta excepto cuando dos de los tres numeros son iguales, y el tercero o bienes igual a ellos o bien es nulo.

Demostracion. Esta expresion es simetrica ya que si permutamos todas las variables entre sı, permaneceinvariante. Podemos suponer, sin perdida de generalidad, a ≥ b ≥ c ≥ 0. Notamos que cuando no sucedeni a = b = c ni que dos terminos sean iguales y el otro nulo, nos encontramos en uno de los siguientescasos:

(a) a > b ≥ c,

(b) a ≥ b > c > 0.

Se tiene

ap(a− b)(a− c) ≥ ap(a− b)(b− c)≥ bp(a− b)(b− c)= −bp(b− a)(b− c).

Ademas, cuando se cumple (a), la primera desigualdad realizada es estricta. Luego

ap(a− b)(a− c) + bp(a− b)(c− b) ≥ 0,

Page 70: Desigualdades matemáticas clásicas · 2018-02-27 · 3 Resumen En esta memoria recogemos algunas de las desigualdades m as importantes en Matem aticas, sus or ge-nes y aplicaciones.

66 CAPITULO 8. LA DESIGUALDAD DE SCHUR

con desigualdad estricta si se verifica (a). Por otra parte,

cp(c− a)(c− b) ≥ 0,

con desigualdad estricta si ocurre (b). Luego

ap(a− b)(a− c) + bp(b− a)(b− c)︸ ︷︷ ︸≥0

+ cp(c− a)(c− b)︸ ︷︷ ︸≥0

≥ 0,

con desigualdad estricta si sucede o bien (a) o bien (b).

8.3. Problemas

Problema 22. Sean a, b, c numeros no negativos tales que a+ b+ c = 1. Demostrad que

a3 + b3 + c3 + 6abc ≥ 1

4.

Solucion. La desigualdad es equivalente a

4(a3 + b3 + c3 + 6abc) ≥ 1.

Por tanto, la desigualdad es equivalente a la desigualdad homogenea

4(a3 + b3 + c3 + 6abc) ≥ (a+ b+ c)3 ∀ a, b, c > 0.

Manipulando, obtenemos la desigualdad equivalente

4a3 + 4b3 + 4c3 + 24abc ≥ a3 + b3 + c3 + 3a2b+ 3a2c+ 3ab2 + 3b2c+ 3ac2 + 3bc2 + 6abc,

que podemos factorizar como

4a3 + 4b3 + 4c3 + 24abc ≥ a3 + b3 + c3 + 3(a2(b+ c) + b2(c+ a) + c2(a+ b)) + 6abc.

Juntando terminos semejantes, obtenemos la desigualdad equivalente

3a3 + 3b3 + 3c3 + 18abc ≥ 3(a2(b+ c) + b2(c+ a) + c2(a+ b)),

y dividiendo entre 3 llegamos a

a3 + b3 + c3 + 6abc ≥ a2(b+ c) + b2(c+ a) + c2(a+ b). (8.1)

Teniendo ahora en cuenta la desigualdad de Schur para p = 1,

a(a− b)(a− c) + b(b− c)(b− a) + c(c− a)(c− b) ≥ 0,

y desarrollando los productos, obtenemos

a(a2 − ab− ac+ bc) + b(b2 − ab− bc+ ac) + c(c2 − ac− bc+ ab) ≥ 0,

que es equivalente a

a3 − a2b− a2c+ abc+ b3 − ab2 − b2c+ abc+ c3 − ac2 − bc2 + abc ≥ 0.

Reordenando los factores convenientemente obtenemos

a3 + b3 + c3 + 3abc ≥ a2b+ a2c+ ab2 + b2c+ ac2 + bc2

y, agrupando adecuadamente y sacando factor comun, llegamos a

a3 + b3 + c3 + 3abc ≥ a2(b+ c) + b2(c+ a) + c3(a+ b),

Puesto que 3 · abc ≥ 0, (8.1) es cierta.La igualdad se alcanza cuando, simultaneamente, abc = 0 y la desigualdad de Schur produce una igualdad.O sea, cuando dos de los tres numeros a, b, c son 1/2 y el tercero es 0.

Page 71: Desigualdades matemáticas clásicas · 2018-02-27 · 3 Resumen En esta memoria recogemos algunas de las desigualdades m as importantes en Matem aticas, sus or ge-nes y aplicaciones.

67

Conclusion

En el momento de decidir que Trabajo Fin de Grado realizar, me interese desde un principio por estetema. Me parecio muy acertada la idea de poder combinar teorıa y practica de manera que ambas partestengan un peso similar en el trabajo.

Este proyecto se basa en las asignaturas de la rama de Analisis que hemos cursado, fundamentalmenteen la asignatura de Analisis Real y Funcional estudiada en cuarto.

En la parte teorica he podido plasmar lo aprendido en esta materia y ampliar mis conocimientos. Ala hora de aprender y demostrar nuevos enunciados he encontrado algunas dificultades derivadas de laabstraccion y complejidad que requiere la comprension de ciertos textos matematicos. Con constancia ycon la ayuda de mi tutor ha sido posible solventar estos obstaculos y poder hacer un buen seguimientode los mismos.

En la parte practica he tenido la oportunidad de familiarizarme con las desigualdades, ya que ha sidoesencial asimilar los nuevos conceptos y manipular nuevos sımbolos y terminos para poder llegar hastala solucion de los problemas. Debido a que una gran parte de estos han sido extraıdos de Olimpiadasdirigidas a estudiantes preuniversitarios, la manera de incorporar nuevas ideas me ha resultado, en loscasos mas sencillos, semejante al modo de aprender a razonar las matematicas en el instituto.

Igualmente interesante considero el hecho de haber podido conocer y aprender un poco mas sobreLATEX, herramienta que encuentro elemental para redactar cualquier escrito cientıfico y en particular, decaracter matematico.

Entre las aplicaciones que hemos tratado, algunas me han hecho reafirmarme en la idea de la relevan-cia que tienen las matematicas en la vida cotidiana, y en especial, las desigualdades. Sirva como ejemploel Problema de la lazada, analizado en el primer capıtulo. Este problema me ha hecho ser consciente de lasconsecuencias inmediatas que tienen las desigualdades en la vida real, como es el caso de la importanciaque puede tener para una empresa del calzado el hecho de poder cumplir sus objetivos minimizando lacantidad de cuerda necesaria en el proceso de fabricacion, y por tanto, aumentando su beneficio.

Y del mismo modo que esta posible aplicacion en el ambito empresarial, las desigualdades matematicasse presentan en innumerables situaciones diarias, a pesar de que en muchas ocasiones pasen desapercibi-das. Por ejemplo, tienen especial interes en areas como la medicina, tecnologıa o economıa, e incluso seponen de manifiesto en la naturaleza del universo. Muchas veces pensamos en las Leyes de la Fısica comoecuaciones (o igualdades). Sin embargo, algunas de ellas, como la Segunda Ley de la Termodinamica, quemide el crecimiento de la entropıa en el universo, se expresa en terminos de una desigualdad.

Page 72: Desigualdades matemáticas clásicas · 2018-02-27 · 3 Resumen En esta memoria recogemos algunas de las desigualdades m as importantes en Matem aticas, sus or ge-nes y aplicaciones.
Page 73: Desigualdades matemáticas clásicas · 2018-02-27 · 3 Resumen En esta memoria recogemos algunas de las desigualdades m as importantes en Matem aticas, sus or ge-nes y aplicaciones.

69

Bibliografıa

[1] 50 anos de Olimpiada Matematica en Espana 1964-2014. Real Sociedad matematica Espanola, firstedition, 2014.

[2] Colin Bennett and Robert Sharpley. Interpolation of operators, volume 129 of Pure and AppliedMathematics. Academic Press, Inc., Boston, MA, 1988.

[3] Radmila Bulajich Manfrino, Jose Antonio Gomez Ortega, and Rogelio Valdez Delgado. Inequalities:a mathematical olympiad approach. Birkhauser Verlag, Basel, 2009.

[4] Jean Dieudonne. Calcul infinitesimal. Methodes. Premiere edition, 1968.

[5] Dusan Djukic, Vladimir Jankovic, Ivan Matic, and Nikola Petrovic. The IMO compendium. Pro-blem Books in Mathematics. Springer, New York, 2006. A collection of problems suggested for theInternational Mathematical Olympiads: 1959–2004.

[6] G. H. Hardy, J. E. Littlewood, and G. Polya. Inequalities. Cambridge, at the University Press, 1952.2d ed.

[7] Thomas J. Mildorf. Olympiad inequalities. https://www.artofproblemsolving.com/articles/files/MildorfInequalities.pdf. Accessed 6-6-2016.

[8] Jose H. Nieto. Desigualdades. http://www.acm.org.ve/desigual.pdf. Accessed 6-6-2016.

[9] Walter Rudin. Real and complex analysis. McGraw-Hill Book Co., New York, third edition, 1987.

[10] Leonardo Urbina. Notas en desigualdades. http://www.acm.org.ve/desigualdades urbina.pdf. Ac-cessed 6-6-2016.

[11] Milivoje Lukic Zoran Kadelburg, Dusan Dukic and Ivan Matic. Inequalities of karamata, schur andmuirhead and some applications. http://elib.mi.sanu.ac.rs/files/journals/tm/14/tm813.pdf. Acces-sed 6-6-2016.